You are on page 1of 209

GRADE 10-12 REVISION QUESTIONS AND

ANSWERS PAMPHLET BASED ON ECZ


PASTPAPERS FROM 2015-2021
1 MOUNT K PUBLISHERS (0979595713)
EXAMINATIONS COUNCIL OF ZAMBIA
Joint Examination for the School Certificate
and General Certificate of Education Ordinary Level
SCIENCE 5124/1
(PHYSICS PART)
PAPER 1 Multiple Choice

Monday 12 OCTOBER 2015 1HOUR

1. The diagram below shows part of the micrometer screw gauge. What is the reading

shown in the diagram?


A. 6.69 mm B. 6.86 mm
C. 6.68 mm D. 5. 68 mm

2 MOUNT K PUBLISHERS (0979595713)


2. The velocity time graph for the motion of a trolley is shown below.

What distance did the trolley travel when there was no resultant force acting on it?
A. 200 m B. 320 m C. 400 m D . 520 m
3. Which of the following quantities changes when a body is accelerating?
A. Mass of the body B. Weight of the body
C. velocity of the body D. force acting on the body
4. To find density of the liquid, a pupil noticed that 90g of the liquid occupied the same
volume as 114g of water of density 1000kg/m3. What was the density of the liquid?
A. 0.79g/cm3 B. 1.27g/cm3 C. 2.04g/cm3 D. 4.90g/cm3
5. A number of forces are acting on a body as shown in the diagram below

What is the magnitude of the resultant force acting on the body?


A. 0N B. 3 N C. 5 N D. 12 N
6. A toy car of mass 600g moves through 6 m in 2s. The average kinetic energy of the toy
car is ….
A. 0.0227 J B. 0.27 J C. 2.7 J D. 66.67 J

3 MOUNT K PUBLISHERS (0979595713)


7. The diagram below shows an inclined plane used to lift a load of 2000 N.

What is the velocity ratio of the inclined plane as a simple machine?


A. 40.0 B. 20 C. 2.0 D. 0.5
8. Which of the following processes shows that particles of matter are in a random
motion?
A. A small drop of oil spreads into a large circular patch on water.
B. Particles of smoke are seen to move haphazardly.
C. Bromine gas spreads slowly into a vacuum.
D. Particles of smoke are seen to move in one direction only.
9. Which of the following describes the difference between x-rays and gamma rays?
A. Gamma rays and x-rays have different sources.
B. C. Gamma rays travel faster than x-rays.
C. Gamma rays have a longer wavelength than x-rays.
D. D. Gamma rays do not affect photographic films.
10. A ripple tank with a vibrator hitting the surface of water with a frequency of 60Hz
produces 10 complete waves in a distance of 15 cm. What is the velocity of the
water waves produced?
A 0.9m/s B. 9m/s C. 90m/s D. 900m/s
11. When a sound wave passes through air, the particles of air ...
A. Oscillate perpendicular to the direction of the wave.
B. Oscillate parallel to the direction of the wave
C. Oscillate up and down
D. Do not move at all.

4 MOUNT K PUBLISHERS (0979595713)


12. A ray of light from a ray box is directed on one side of a parallel sided glass block.
Which diagram shows the path of the ray of light through the glass block?

13. The diagram below shows a soft iron rod placed inside a solenoid connected to a
d.c supply. The ends of the soft iron rod are marked X and Y.

When current is switched on which of the following occurs?


A. X attracts pieces of iron filings.
B. X becomes the South Pole.
C. Y becomes the North Pole
D. Y does not attract pieces of iron filings.
14. The diagram below shows a charged polyethene rod which is moved near to end
R of a metal sphere mounted on an insulating stand?

Which of the following shows the correct charge distribution on the sphere?

5 MOUNT K PUBLISHERS (0979595713)


15. A battery moves a charge of 60C around a circuit at a constant rate in 20
seconds. What is the current flowing in the circuit?
A 80A B 40A C 3.0A D 0.3A
16. A heater is connected to a 200V supply. If the heating element has a resistance of
10Ω, how much electrical energy is supplied to the heater in 2 minutes?
A 800 000J B 480 000J C 8 000J D 4 000J
17. The diagram below shows an ideal transformer which is supplied with an
alternating voltage of 240V and a current of 1A.

If the secondary current is 5A, what is the secondary voltage?


A 1400V B 1200V C 240V D 48V
18. What is the purpose of X-plates and Y-plates in a Cathode Ray Tube?
X – Plates Y – Plates
A Deflecting electrons horizontally Deflecting electrons vertically
B Deflecting electrons vertically Deflecting electrons horizontally
C Accelerating electrons Deflecting electrons vertically
D Deflecting electrons horizontally Accelerating electrons

19. Which of the following radiations is NOT deflected by an electric field?


A Alpha B Beta C Gamma D Proton
20. 90
The radioactive nucleus 38 (strontium) undergoes beta decay.
Which of the following is the correct nuclear equation?

6 MOUNT K PUBLISHERS (0979595713)


EXAMINATIONS COUNCIL OF ZAMBIA
Examination for General Certificate of Education Ordinary Level
SCIENCE 5124/1
PAPER 1 (PHYSICS PART)

Monday 11 JULY 2016 Time: 1 Hour

1 What is the correct scientific notation of a quantity 157nm?


A 1.57 x 109m B 1.57 x 107m
C 1.57 x 10-9m D 1.57 x 10-7m
2 The diagram below shows how a Post Bus was moving.

What was happening to the bus at time intervals of 5s —10s, 10s — 30s and 30s — 50s?
5s —10s 10s — 30s 30s — 50s
A Reducing in speed Moving at constant speed Increased speed
B Increased speed Stopped Started moving
C Going down a slope Moving at constant speed Climbing a hill
D Increased speed Stopped Reduced speed
3 Which one of the following is the correct weight of a 50kg bag of maize on both the
moon and the earth?
Moon Earth
A 0.85N 50N
B 8.5N 50N
C 85N 500N
D 850N 500N

7 MOUNT K PUBLISHERS (0979595713)


4 A 125g wooden plank was placed in a beaker, and below was the reading before and
after placing it in the beaker.

What is the density of the wooden block?


A 0.1 g/cm3 B 0.5 g/cm3
C 1.0 g/cm3 D 5.0 g/cm3
5 A 4kg brick is dropped from the top of a building whose height is 30m. The brick reaches
the ground with a velocity of
A 1200 m/s. B 43.6 m/s.
C 40 m/s. D 34.6 m/s.
6 A machine with a velocity ratio of 6 requires 800J of work to raise a load of 60kg through
a vertical distance of 1m.
Find the mechanical advantage of the machine.
A 0.45 B 0.75
C 4.5 D 75.0
7 The common understanding of energy conservation is that it ...
A can be transformed from one form to another.
B remains the same in all forms.
C can be destroyed.
D can be created.
8 The higher an object is from the ground, the ...
A more potential energy it has. B higher its centre of mass is.
C lower its centre of mass is. D more unstable it is.

8 MOUNT K PUBLISHERS (0979595713)


9 In order to create a temperature scale, two fixed points are needed; the ice point and the
steam point. Which of the following is used to determine the ice point? The
temperature...
A at which sea water freezes.
B at which snow melts.
C of ice in a freezer.
D of melting point of pure ice.
10 It was discovered that 120 crests passed through a slit in two hours.
What is the frequency of this wave?
A 100Hz B 60Hz
C 2Hz D 1Hz
11 Which type of the electromagnetic radiation has the longest wavelength?
A Infra-red rays B Radio waves
C Ultraviolet rays D X-rays
12 Which of the following correctly gives the properties of the sound waves?
Nature Speed in air
A Longitudinal 340m/s
B Longitudinal 3.0 x 108m/s
C Transverse 340m/s
D Transverse 3.0 x 108m/s
13 The process of removing magnetic properties from a magnet is known as
A attraction and repulsion. B demagnetisation.
C induction. D magnetisation.
14 An electric bulb is rated 1,200 watts. If the current flowing in the filament of the bulb is
5A, what is the resistance of this bulb?
A 240 ohms
B 96 ohms
C 48 ohms
D 28 ohms

9 MOUNT K PUBLISHERS (0979595713)


15 The diagram below shows a circuit with an ammeter and two resistors connected.

If the ammeter reading is 3A, what is the value of resistor R1?


A 4Ω B 6Ω C 12Ω D 18Ω
16 Which part of the Cathode Ray Oscilloscope (CRO) emits electrons by thermionic
emission?
A. Cylindrical anode B. X-plates C. Cathode plate D. Y-plates
17 The diagram below shows a transformer.

Assuming the transformer is 100% efficient, what is the size of the electric current in the
primary coil?
A 5.0A B 2.5A C 0.2A D 0.05A
89 85
18 A radioactive element Yttrium 39 decays to isotopes rubidium 37 according to
the decay equation.
89
39 − −→ 85 37 +
What could be X?
A Gamma radiation B Beta particle
C Alpha particle D Both alpha and Gamma particles
19 A radioactive substance has a mass of 600g and a half-life of 12 years.
How much of this substance decays after 36 years.
A 75g B 125g C 475g D 525g
20 Which of the following statements correctly shows the name of the nuclear radiation and
its use?
Name of radiation Use of nuclear radiation
A Alpha particle Treatment of cancer
B Gamma rays Monitoring paper thickness
C Gamma rays Treatment of cancer
D Beta particle Sterilizing instruments
10 MOUNT K PUBLISHERS (0979595713)
EXAMINATIONS COUNCIL OF ZAMBIA
Examination for General Certificate of Education Ordinary Level
SCIENCE 5124/1
PAPER 2 (PHYSICS)

Monday 25 JULY 2016 Time: 1 Hour 15 Minutes

SECTION A [45 MARKS]


1. The figure below shows an experiment which was carried out to measure the time
interval of a simple pendulum.

The bob was oscillating between A and C.


(a) State the meaning of the term ‘Oscillation’. [1]
(b) If the pendulum bob took 0.20s to swing from A to C, calculate the period. [2]
(c) Using your answer in (b), determine the frequency of the bob. [2]

2. A lion of mass 200kg is transferred from the moon to Earth. Acceleration of free fall on
the moon is 1.67m/s2 while that on Earth is 10m/s2.
(a) state
(i) An instrument commonly used in the laboratory for measuring the
amount of material in a body. [1]
(ii) The mass of the 200kg lion on Earth. [1]
(b) Calculate the weight of the lion on Earth. [2]

11 MOUNT K PUBLISHERS (0979595713)


3. (a) What is meant by the ‘moment’ of a force about a point? [1]
(b) Moment of a force about a point depends on two factors; what are they?
[1]
(c) The figure below shows a uniform metre rule which is pivoted at the 20cm mark
and balanced horizontally by an object of weight 3.25N placed at the 10cm mark.

Calculate the weight of the metre rule. [2]


4. The figure below shows a stone of mass 2kg which drops from the top of a cliff and takes
two seconds to strike the ground. Acceleration of free fall, g = 10m/s2.

(a) Name the form of energy possessed by the stone before it falls. [1]
(b) Determine the height h of the cliff. [2]
(c) Calculate
(i) The kinetic energy of the stone when half way down. [2]
(ii) The final velocity of the stone as it strikes the ground. [2]

5. Two metal plates A and B, one with shiny surfaces and the other painted black
are heated to a temperature of 100°C. The metal plates are then placed in the
open air or insulating materials and allowed to cool. The table below shows the
results obtained with time.

12 MOUNT K PUBLISHERS (0979595713)


Time in seconds Temperature of metal plate in oC
A B
0 100 100
30 92 85
60 87 74
90 84 67
120 81 63
150 79 60

(a) By what process (es) of heat transfer are the metal plates losing heat to
the surrounding?
(b) Which metal plate is losing heat faster?
(c) State, with a reason which metal plate A or B has shiny surfaces.

6. Figure 61 shows a vibrating tuning form, Y in air.

(a) State the nature of wave produced by the tuning fork. [1]
(b) Show diagrammatically the regions created by the wave produced, as it
passes through the air.
(c) Explain what happens to the speed of the wave in (a) if the rate of
vibration of Y increases but the length of the wave produced remains
the same.

13 MOUNT K PUBLISHERS (0979595713)


7. Figure 7.1 shows a ray of light entering and leaving a glass block.

(a) Name the rays labelled A to D


(b) On the diagram, indicate i and r for angles of incidence and refraction
respectively. [1]
(c) For a certain piece of glass block the angle of incidence was found to be
25° and angle of refraction was found to be 16°. Using this information,
Calculate the refractive index for this piece of glass block to two decimal
places.
8. Figure 8.1 shows a metal bar placed inside a vertical solenoid, above a small pile
of iron nails.

Describe the behavior of the iron nails when the switch above is on for
several seconds, then off, if the metal is made from...
(i) Aluminium [1]
(ii) Iron [2]
(iii) Steel [2]
14 MOUNT K PUBLISHERS (0979595713)
9. Figure 9.1 shows a stream of α, β and γ rays passed through a uniform electric
field represented by tracks A, B and C.

(a) Identify the tracks marked A, B and C representing the radiations.


(b) What is the charge for the radiation represented by the track marked B?
(c) Thorium nucleus undergoes alpha-decay by emitting one particle to form
a daughter element whose symbol is Rn.
232
90 ℎ → + ….
Complete the equation above that represent the decay of the Thorium
nucleus; include the nucleon and proton numbers of the particles
concerned. [2]

SECTION B [20 Marks]


10. (a) State two effects that a simple machine can do to make work easier for a man.
(b) Explain why the efficiency of a simple machine
(i) is normally less than 100% [1]
(ii) Cannot exceed 100% [1]
(c) Draw a labeled diagram to show a block and tackle pulley system of velocity ratio 4.
(d) John uses the machine in 29 (c) to raise a 60N load from the ground to a distance
of 0.4m. The effort used by John to do the work is 20N. Calculate:
(i) The mechanical advantage and [2]
(ii) Efficiency of the machine [2]

15 MOUNT K PUBLISHERS (0979595713)


11. (a) Distinguish between transverse and longitudinal waves.
Include one example of each type of wave.
(b) Describe an experiment to show reflection of water waves. [5]
(c) Figure 11.1 shows a displacement-time graph for a water wave moving
in the direction x.

(i) State the period of the wave [1]


(ii) If the speed of the water wave is 0.2m/s, calculate the
wavelength. [2]
12. (a) State the difference between 'Electric charge and electric current.
(b) Below is a table of results obtained from an experiment to determine the
relationship between the voltage and current through a metallic
conductor.

Voltage (v) 0 2 4 6 8 10 12
Current (A) 0 0.5 1.0 1.5 2.0 2.5 3.0

(i) Plot a graph of voltage against current [3]


(ii) Calculate the resistance of the metallic conductor [2]
(c) Figure 12.1 shows 4Q and 6Q resistors connected in parallel.

i. Find the effective resistance [2]


ii. Determine the current in the circuit. [2]

16 MOUNT K PUBLISHERS (0979595713)


Centre Candidate
Number Number
Candidate Name

EXAMINATION COUNCIL OF ZAMBIA

Examination for School Certificate Ordinary Level

Science 5124/1
Paper 1

Thursday 5 NOVEMBER 2016

SECTION A [20 MARKS]


Answer all the questions on the answer grid provided.
1 The diameter and the length of a thin wire approximately 50cm in length, is
measured as precisely as possible. What are the best instruments to use?
Diameter Length
A Micrometer screw gauge Rule
B Micrometer screw gauge Vernier calipers
C Rule Tape
D Vernier calipers Rule

2 Which of the following is the S.I unit for time?


A Millisecond
B Second
C Minute
D Hour

3 The diagram below shows a speed time graph for a car.

17 MOUNT K PUBLISHERS (0979595713)


At which points is the car moving with zero acceleration? It ls at points …
A A and D.
B B and C
C A and C.
D B and D.

4 When a nut and bolt are difficult to undo, It is easier to tun the nut by using a longer
spanner because the longer spanner gives …
A less friction.
B more friction.
C a larger turning effect.
D a smaller turning effect.

5 A body moving with a speed of 30m/s has kinetic energy of 1800J. What is its mass?
A 120kg
B 60kg
C 4Kg
D 2kg

6 Which of the following properties of a body is affected by a change in gravitational field?


A Mass
B Temperature
C Volume
D Weight

7 Which substance A, B, C or D below is a liquid at 20°C?


Substance Melting point/°C Boiling point/°C
A -218 -183
B -39 357
C 39 444
C 44 280

8 A boy strikes a rail with a hammer to create a sound along the rail. A girl listens
with her ear against the rail. One second later, the girl hears a sound through the air.

18 MOUNT K PUBLISHERS (0979595713)


How long will it take for the sound to reach the girl through the rail?
A 0 seconds
B More than 1 second
C 1 second
D Less than 1 second

9 Which of the following waves belong to the same class?


A Light, radio and water nipples
B Sound, radio and light
C Microwaves, infra red and sound
D Gamma, sound and radio

10 Water waves are produced with a frequency of 5Hz by hitting the water surface
with the tip of a pencil. If the water travels 30m in 6s, what is the wavelength or
the wave?
A 0.5m
B 1.0m
C 1.5m
D 2.0m

11 The diagram below shows a beaker of water containing à red crystal which slowly
dissolved. Gentle heat was applied below the crystal.

The red colour is seen to rise. What Is the name of this process?
A Conduction
B Convection
C Evaporation
D Radiation

19 MOUNT K PUBLISHERS (0979595713)


12 Which characteristics describe an image formed by a plane mirror?
A Real and inverted
B Virtual and upright
C Real and larger than the object
D Virtual and smaller than the object

13 The diagram below shows a ray of light directed at a plane mirror.

What is an angle of incidence and the angle of reflection?


Angle of incidence Angle of reflection
o
A 40 C 40oC
B 40oC 50oC
C 50oC 40oC
D 50oC 50oC

14 Which row describes the case with which Iron or steel can be magnetized and
demagnetized?
Metal Magnetised Demagnetised
A Iron Easy Easy
B Iron Easy Difficult
C Steel Difficult Easy
D Steel Difficult Difficult

15 A permanent magnet is placed on a flat horizontal surface. A plotting compass is placed


on the dot as shown in the diagram below.

20 MOUNT K PUBLISHERS (0979595713)


In which direction will the plotting compass point, ignoring any effect of the
earth's magnetic field?

16 A positively charged rod is held close to, but not touching an insulated metal sphere.

Which diagram shows the charges that are induced on the sphere?

17 A 60V battery is connected to four resistors as shown in the diagram below.

What is the energy dissipated by the 4Ω resistor in 2 seconds?


A 300J
B 450J
C 600J
D 750J

21 MOUNT K PUBLISHERS (0979595713)


18 The diagram below shows a horizontal beam of electrons entering a region between
two charged plates.

In which direction is the electron beam deflected?


A Downwards
B Into the page
C Out of the page
D Upwards

19 A sample contains 12 000 radioactive atoms of a particular nuclide. After undergoing


two half-lives, how many atoms have decayed?
A 9000
B 6000
C 3000
D 0

20 A nucleus is represented by 23091 . It emits one alpha particle followed by two beta
particles. What is the resulting nucleus?

226
A 91

226
B 87

230
C 90

230
D 91

Section B (45 marks]


Answer all questions in this section.
Write your answers in the spaces provided on the question paper.

22 MOUNT K PUBLISHERS (0979595713)


1 Figure 1.1 shows a measuring device.

(a) Name this measuring device.

(b) What ls the reading shown on the device?

(c) State two precautions to be taken when using this device


[Total: 4 marks]

2 A stone of mass 1.0kg is dropped from a certain height and takes 8 seconds for
the stone to strike the sandy ground. Upon reaching the ground, the stone
penetrates 5.0cm into the ground. (Take g = 10m/s2).

(a) Calculate the height from where the stone is dropped.

(b) Calculate the kinetic energy of the stone on reaching the ground.

(C) Calculate the average retarding force.


[Total: 6 marks]

3 In a certain town when the temperature was 27°C and the atmospheric pressure
was 99000Pa, a 300cm3 container was sealed. The container which contained only
air was placed into a refrigerator until its temperature cooled to 5°C. Calculate the
new pressure in the container.

[Total: 3 marks]

23 MOUNT K PUBLISHERS (0979595713)


4 Figure 4.1 shows a pulley system used to lift a load of 9600N using an effort of 2400N
through a vertical distance of 20m.

(a) Calculate the mechanical advantage of the above pulley system.

(b) What is the velocity ratio?

(c) What is the efficiency of the pulley system?

(d) Calculate the distance moved by the effort


[Total: 6 marks]

5 Figure 5.1 and figure 5.2 show the displacement-distance and the displacement-time
graphs respectively. The graphs are for a water wave in a ripple tank.

24 MOUNT K PUBLISHERS (0979595713)


(a) State the wavelength of the wave.

(b) State the period of the wave.

(c) What is the speed of the water wave?


[Total: 4 marks]

6 Figure 6.1 shows a laboratory thermometer.

Figure 6.1

(a) State the range of the thermometer.

(b) Describe how the behaviour of a more sensitive thermometer is different


from a less Sensitive thermometer

(c) State one change in the design of the thermometer to make k more
sensitive.

(d) Describe how a clinical thermometer differs from a laboratory thermometer.


A diagram may be included in your answer
[Total: 6 marks]

7 Figure 7.1 shows a 6.0V power supply connected to two resistors which are in parallel,

25 MOUNT K PUBLISHERS (0979595713)


(a) Calculate the effective resistance of the two resistors.

(b) Calculate the value of the current in

(i) the 4Ω resistor

(ii) the battery

(c) Would the addition of an extra resistor between A and B, in parallel with
the original resistors, Increase, or decrease the current in the battery?
Explain your answer
[Total: 8 marks]

8 Figure 8.1 shows a transformer which has a primary coil of 500 turns and a secondary
coil of 25 turns.

(a) Calculate the secondary voltage of the primary coil if connected to 240OV
mains Supply.

(b) Calculate the efficiency of the transformer if the primary current is 0.6A
and the secondary current is 8A
[Total: 4 marks]

26 MOUNT K PUBLISHERS (0979595713)


9 Lanthanum -139 (13957 ) emits an alpha particle to form caesium (Cs) followed
by an emission of a beta particle to form the nucleus of barium (Ba). Express
these changes using nuclear equations.

(a) Emission of alpha particle

(b) Emission of beta particle


[Total: 4 marks]

Section C [20 marks]


Answer any two (2) questions from this section in the Answer Booklet provided.

1 (a) Describe an experiment to determine the position of the centre of mas


of an irregularity shaped piece of a cardboard. [6]

(b) A uniform metre rule is balanced at the 30cm mark with a load of 0.80N is hung
at the zero mark as shown in figure 10.1

Figure 10.1

(i) At what point on the rule is the centre of mass of the rule.

(ii) Calculate the weight of the rule [3]

[Total: 10 marks]

27 MOUNT K PUBLISHERS (0979595713)


2 (a) When a suitable counter was placed near a radioactive source of B-particle, the
following rates of emission were obtained at the times shown in figure 11.1.

Times/minutes 0 5 10 15 20
Count rate/counts in seconds 295 158 86 47 25
Figure 11.1

(i) Plot a graph of count rate (x-axis) against time (y-axis)

(ii) Determine the half-life of the source. Show clearly how you obtain this.

(c) A radioactive isotope, radon-220(Ra) decays by the emission of an -particle.


The half-life of this Isotope is 1 minute. What mass of radon-220 will decay to a
mass of 1.5g in a time of 5 minutes?

(d) In some industrial situations, leaks in lead pipes are found by pumping a solution
of radioactive isotopes through the pipes.

(i) Give one advantage and one disadvantage of using α-Source and
emitters in this task respectively.

(ii) What kind of an Isotope in terms of half-life would you use for the task
mentioned above? State whether the isotope should have a half-life of
one year, one month, one week, one day or one minute.

[Total: 10 marks]

28 MOUNT K PUBLISHERS (0979595713)


Centre Candidate
Number Number

Candidate Name

EXAMINATION COUNCIL OF ZAMBIA

Examination for General Certificate of Education Ordinary Level

Science 5124/1
Paper 1

Monday 31 JULY 2017

Time 2 hours

SECTION A [20 MARKS]


Answer all the questions on the answer grid provided.
A1. The diagram below shows part of the ruler used to find the length of the nail.

What is the length of the nail?


A. 2.2 cm B. 2.7 cm C. 3.2 cm D. 3.7 cm
2. The diagram below shows a block of wood of density 0.6g/cm3.

What is the mass of the block?


A. 30g B. 50g C. 300g D. 500g

3. The diagram below shows a graph of how a distance covered by a woman varies with
time as she takes a walk from her home to the market.

29 MOUNT K PUBLISHERS (0979595713)


What was happening in the region AB during the woman’s walk? She ..
A. Walked with a constant B. walked faster than before
C. walked slower than before D. stopped walking
4. A parachutist of mass 60kg falls with a constant velocity of 5m/s together with a
parachute of mass 20kg. Taking g to be 10N/kg what is the resultant force on the system?
A. 0N B. 400 N C. 600 N D. 800 N
5. The diagram below shows a frictionless pulley used to lift an 8000 N block of concrete.

What is the minimum effort required to raise the block?


A. 1600N B. 2000N C. 3600N D. 8000N

30 MOUNT K PUBLISHERS (0979595713)


6. The diagram below shows a model of a crane with a counter balance weighing 200N. This
counter balance can be moved further or closer to 0 to accommodate different loads.

What is the maxmum load the crane can safely lift?


A. 200N B. 400N C. 600N D. 1000N
7. A dog running at constant speed of 3m/s increases its speed to 7m/s upon seeing a lion.
If the mass of the dog is 20kg, the work it does in achieving the new speed is
A. 40 J B. 160 J C. 400 J D. 580 J
8. A thermos flask contains a vacuum. What is the purpose of this vacuum? Prevents
A. conduction and radiation.
B. conduction and convection.
C. radiation and convection.
D. conduction, convection and radiation.

9. Molecules of a liquid evaporate from a container and the temperature of the liquid
left in the container changes. From which part of the body of the liquid do the
molecules escape and what is the effect on the temperature of the liquid left in the
container?
Molecules escape from Temperature of liquid left in container
A. All parts of the liquid Decreases
B. All parts of the liquid Increases
C. Only the liquid surface Decreases
D. Only the liquid surface Increases

31 MOUNT K PUBLISHERS (0979595713)


10. The diagram below represents a transverse wave.

Between which two marked points is the distance equal to the wavelength of
the wave?
A. a and d B. c and g C. b and g D. e and f
11. A lighted candle is placed in front of a loudspeaker that produces a loud,
steady sound at regular intervals.

What type of wave is produced by the speaker and in which direction does it cause the
flame to tilt?

12. The diagrams below show three rays of light incident on the boundary
between a glass block and air. The angles of incidence are different.

What is the possible critical angle?


A. 60° B. 45° C. 30° D. 15°

32 MOUNT K PUBLISHERS (0979595713)


13. The diagram below shows an object placed under water and being viewed from
the top

What is the refractive index of water?


A. 1.31 B. 1.50 C. 1.60 D. 1.60
14. The diagram below shows a steel magnet being withdrawn from a coil of wire in the
direction shown to a point as far away as possible.

What is the effect on the steel magnet, due to this action?


A. No change B. Becomes magnetized
C. Becomes demagnetized D. Becomes a stronger magnet
15. The diagram below shows an illustration of a transformer with 100 turns
on the primary coil and 25 turns on the secondary coil.

What is the voltage induced across the secondary coil?


A 3.0V B 4.0V C 48V D 300V

33 MOUNT K PUBLISHERS (0979595713)


16. The diagram below shows a positively charged rod placed close to two metallic spheres A
and B which are initially neutral. Which statement is correct? A has ...

A lost positive charges to sphere B.


B gained electrons from sphere B.
C has gained electrons from the charged rod.
D has lost positive charges to the charged rod.
17. The diagram below shows a circuit with four ammeters connected at different
positions and three resistors of different values. Which of the four ammeters
labelled A, B, C or D would show the largest reading?

18. A Secretary in a Manager's office uses the following appliances with respective
power ratings.
1 Electric kettle — 1200W 2 Printer — 600W 3 Computer — 100W
Given that the cost of electricity is K0.80/kWh, calculate the cost of operating
all the three appliances at once for 12 hours.
A. K 5.60 B. K8.00 C. K 16.30 D. K 18.24
19. Which of the following is correct about the purpose of X-plates and Y-plates in a
Cathode Ray Oscilloscope?
A. Deflecting the electron beam vertically
B. Deflecting the electron beam horizontally
C. Deflecting the electron beam vertically and horizontally
D. Increasing the speed of the electron beam towards the screen.

34 MOUNT K PUBLISHERS (0979595713)


20. The diagram below shows a box used for storing radioactive sources.

Which material is best for lining the box to prevent the escape of most
radioactive emissions?

A Lead B Steel C Copper D Aluminium

SECTION B [45 Marks]


1. The figure below shows part of vernier calipers.

(a) What is the reading of the vernier calipers? [2]


(b) Write in words the SI units of the following physical quantities and state their symbols.
SI unit in words SI unit in symbol
(i) Velocity… .................................................................................. [1]
(ii) Temperature .............................................................................. [1]
(iii) Acceleration ............................................................................... [1]
2. (a) What is the difference between density and relative density? [1]
(b) The diagram below shows a cuboid container that has a 5cm square base and contains
water to a height of 6cm.

35 MOUNT K PUBLISHERS (0979595713)


(i) What is the volume of the water? [1]
(ii) A stone is immersed into the water in the cuboid causing the water to rise to a
height of 8cm. determine the volume of the stone. [2]
(iii) If the mass of the stone is 80kg, calculate the density of the stone. [2]
3. The figure below shows a ramp being used to lift a box weighing 480N through a distance
of 3 meters and height of 1 meter by applying a force F of 200N.

(a) State the meaning of the term ‘simple machine’. [1]


(b) Calculate the mechanical advantage of the ramp shown above. [2]
(c) Calculate the efficiency of the ramp. [2]

4. Figure B4.1 shows a glass syringe with a sealed tip containing a gas at an initial
pressure of 360Pa placed in hot water. After a few minutes the piston in the
syringe moved up.

36 MOUNT K PUBLISHERS (0979595713)


(a) Using the kinetic theory explain why the piston in the syringe moved
upwards when the syringe was placed in hot water.
(b) The piston was pushed downwards to 20cm3 while the temperature was
kept constant.
(i) In terms of the kinetic theory explain why the pressure of the gas
in the syringe increases.
(ii) Calculate the pressure of gas in the syringe.
5. Figure B5.1 shows various Regions of the Electromagnetic Spectrum.
K L M Visible light Ultra violet N O

(a) What is the name of component N?


(b) Give one practical use of Region M.
(c) State one property which is the same for all Electromagnetic Waves.
(d) State one possible source of the radiation from Region O.
6. Figure B6.1 shows a converging lens of focal length 2.0cm used to produce
an image of an object 2.0cm tall placed 5cm from the lens . Using the information
given above,

37 MOUNT K PUBLISHERS (0979595713)


(i) Draw to scale on figure B6.1, a ray diagram to locate the image formed.
(ii) Calculate the magnification of the image formed.
7. Figure B7.1 shows magnetic field lines between the poles of two magnets.

(a) Name point R.


(b) If P is a south pole what are the poles Q and T?
(c) Explain how soft iron keepers help magnets retain their magnetism for a
longer period of time.
(d) State two differences between iron and steel as magnetic materials.
8. Polonium -210(210 84 ) can undergo radioactive decay by emitting an alpha particle to
form lead (Pb).
(a) What is an alpha particle? [1]
(b) Write the equation for the radioactive decay of polonium — 210. [2]
(c) State one use of alpha radiation. [1]
9. Figure 88.1 shows three resistors connected to a 12V battery.

Calculate:
(a) the effective resistance between X and Y [2]
(b) the current flowing in the battery. [2]
(c) the current flowing through the 6Ω resistor. [2]

38 MOUNT K PUBLISHERS (0979595713)


SECTION C [20 Marks]
1. An automobile of mass 2000kg started from rest position and increased its
speed uniformly to 9m/s in 30s. It maintained this speed for another 500s.
(a) Sketch the speed time graph of the automobile for the journey described.
(b) Calculate the uniform acceleration of the automobile. [2]
(c) How much force was required to produce this acceleration? [1]
(d) Calculate the distance covered by the automobile when it was moving at constant speed. [2]
(e) What was the average speed of the automobile for the journey described?

2. Table C2.1 shows a table with corresponding values of potential difference


across a torch bulb and the current flowing through the same torch bulb.
Pd/Volts 0.20 1.00 5.00 10.00 16.00 23.00 31.00 40.00
Current/Amperes 0.40 0.80 1.20 1.60 2.00 2.40 2.80 3.20

(a) Using correct circuit symbols, draw a clearly labelled diagram of a circuit
which could have been used to obtain this data. [2]
(b) On the graph paper provided, and using the data in the figure above, plot
a graph of the current on the X-axis and the p.d. on the Y-axis. [3]
(c) From the graph state whether or not the filament of the torch bulb is an
ohmic conductor. Explain your answer. [2]
(d) (i) Use the graph you have drawn to determine the value of
the p.d across the torch bulb when the current flowing through it is 2.6A
(ii) Calculate the resistance of the torch bulb when the current
through it is 2.6A. [2]
3. A radioactive element 22398 emits one Beta particle followed by 2 Beta particles.
(a) What is a Beta particle? [1]
(b) Write the nucleon number and proton number of the remaining nuclide
after the two emissions. [2]
(c) A 400g radioactive sample has a half-life of 4 years.
(i) On a graph paper plot a graph to show this decay curve after the
period of 32 years. [5]
(ii) What period of time would it take for the sample to reduce to 40g? [1]
(iii) Name a source of gamma radiation. [1]

39 MOUNT K PUBLISHERS (0979595713)


Centre Candidate
Number Number
Candidate Name

EXAMINATION COUNCIL OF ZAMBIA

Examination for School Certificate Ordinary Level

Science 5124/1
Paper 1

Wednesday 8 NOVEMBER 2017

SECTION A [20 MARKS]


Answer all the questions on the answer grid provided.
A1 Which of the following instruments can best be used to measure the internal diameter of
a pipe?
A Meter rule B Vernier calipers
C Engineers calipers D Micrometer screw gauge.
A2 A stone has a mass of 390g and a density of 2.7g/cm3, Cooking oil has a density of
0.90g/cm3. What mass of oil has the same volume as the stone?
A 130g B 160g C 900g D 1200g
A3 The diagram below shows an extract of a tape from a ticker tape vibrator that was
connected to 50Hz a.c power supply.

40 MOUNT K PUBLISHERS (0979595713)


Determine the time taken between points A and B?
A 0.02s B 0.20s C 0.50s D 5.00s
A4 The diagram below shows a box of mass 2.5kg placed on a table.

What force is exerted on the table by the box?


A 2.SN B 10.0N C 25.0N D 250.0N
A5 The diagram below shows a wheel barrow carrying a load of 100N, by apply an effort of
20N.

What is the mechanical advantage of the wheel barrow?


A 0.2 B 5.0 C 20 D 50
A6 The diagram below shows a beam pivoted at one end and a force of 5.0N acting on it
vertically.

If the beam is in equilibrium, what is the weight of the beam?

41 MOUNT K PUBLISHERS (0979595713)


A 5.0N B 3.3N C 3.0N D 2.0N
A7 The diagram below shows a Water reservoir used to run a power station 500m below it
as shown.

If the mass of the body of water in the reservoir is 1011kg. How much gravitational
potential energy is stored by the water in the reservoir?
A (10 11 x 500)J B [(1011 x 10)/500]J
C [1011 x 500)/10]J D (1011 x 10 x 500)J
A8 Which statement about thermal radiation is correct? It ...
A is infra-red radiation.
B involves movement of atoms.
C can only occur in a vacuum.
D involves movement of electrons through a material.
A9 The diagrams below show a mercury thermometer whose bulb is immersed in” pure
melting ice and then in steam from boiling water.

The columns of mercury are 3cm and 23cm in the ice and steam, respectively. What is
the temperature when the column of mercury is 8cm long?
A 20°C B 25°C C 35°C D 40oC
42 MOUNT K PUBLISHERS (0979595713)
A10 A wave of frequency 1000Hz travels between two points P and Q with a velocity of
330m/s. What is the wavelength of the waved?
A 0.33m B 3.03m C 6.70m D 330,000m
A11 Three vibrating objects P, q and R produce waves in air of different frequencies as shown
in the table below.
Object Frequency/Hz
P 25
Q 1000
R 15 000

Which of the above objects produce sound that can be heard by the human ear?
A P and Q B P and R
C Q and R D P, Q and R
A12 The diagram below shows a ray of light directed to a plane mirror at 90°.

What Is the angle of incidence and angle of reflection for the incident ray AO?
Angle of incidence Angle of reflection
A 90° 90°
8 90° 0°
C 0° 90°
D 0° 0°
A13 Of the statements given below, which one is correct?
A X-rays have a lower speed in air than visible light.
B Microwaves have a longer wavelength than visible light.
C Infra-red waves have a lower frequency than radio waves.

43 MOUNT K PUBLISHERS (0979595713)


D Gamma rays have a longer wavelength than ultra-violet rays.
A14 The diagram below shows a step-down transformer.

What is the size of the current flowing in the primary coil if the bulb is working at full
normal brightness?
A 0.25A B 0.40A
C 0.80A D 1.30A
A15 A metal wire is placed between the poles of a magnet. The wire can be moved in each of
the three directions OP, QR and ST.

In which direction or directions must the wire be moved to induce an e.m.f across the
ends of the wire?
A ST
B OP
C OP and ST
D ST and QR
A16 The diagram below shows an electric circuit with an unknown resistor marked X.

44 MOUNT K PUBLISHERS (0979595713)


Calculate the value of resistor X?
A 5Ω B 7Ω C 8Ω D 15Ω

A17 The diagram shows an incomplete circuit.

Four wires of different lengths and thicknesses are connected in turn between points X
and Y. if the switch is closed, which of the following wires will cause the greatest reading
on the ammeter?
A Long and thick
B Long and thin
C Short and thick
D Short and thin
A18 Paul has a 2KW heater. The ZESCO electric metre at his house shows that there is only
3KWh units of electricity left. How long will he run his electric heater before the units run
out?
A 1 hour 15 minutes B 1 hour 25 minutes

45 MOUNT K PUBLISHERS (0979595713)


C 1 hour 30 minutes D 1 hour 45 minutes
A19 A radioactive element 22286 emits two alpha particles, two beta particles and four
gamma particles. What will be the final composition of the nuclide formed?
Proton Neutron Electron
A 82 132 82
B 130 84 84
C 84 130 84
D 83 131 83

A20 The diagram below shows a decay curve for a radioactive substance.

What is the half-life of the substance?


A. 10 minutes B. 20 minutes C. 30 minutes D. 40 minutes
Section B [45 marks]
Answer all questions in this section.
Write your answers in the spaces provided on the question paper.
B1 Figure B1.1 shows a simple pendulum.

46 MOUNT K PUBLISHERS (0979595713)


If the bob of the pendulum takes 0.S seconds to swing between extreme positions A and C.
(a) Calculate the period of oscillation.
(b) Calculate the frequency of the pendulum.
(c) State whether the frequency of oscillation will increase, decrease or remain the
same if:
(i) the length of the string ‹s increased.
(ii) the mass of the bob is increased.
B2 Figure B2.1 shows a distance time graph for a car travelling along a straight road.

(a) Describe the motion of the car between points C and D.


(b) Between which points is the car accelerating? [1]
(c) (i) State the distance travelled by the car between A and B.
(ii) Calculate the average speed of the car between A and D.
B3 An 8N force stretches a spring to the 20cm mark on a metre rule. If the extension on the
spring is 4cm, calculate the;
47 MOUNT K PUBLISHERS (0979595713)
(a) original length of the spring.
(b) extension produced by a 10N force.
B4 An electrician of mass 50kg is fixing a bulb in the corridor on the 20” floor of a building
100m from the ground. make g = 10m/s2]
(a) Determine the gravitational potential energy of the electrician when on the 20th
floor.
(b) While working, the electrician slipped and fell directly to the ground floor. State
with a reason the amount of kinetic energy the electrician possessed as his body
just touched the ground floor.
(i) K.E: …………………………………………………………………………………………
(ii) Reason: ……………………………………………………………………………..
(c) Calculate the time taken for the electrician to fall from the 20” floor to the ground
floor.
B5 The liquids in five Liquid-in-glass thermometers A, B, C, D and E expand linearly with
temperature. All the thermometers have scales marked in °C.
Figure B5.1 accurately represents the scales of these five thermometers.

(a) Use the information from the scales of the thermometers in figure B5.1 to;
(i) State which thermometer has the greatest range.
(ii) State and explain which thermometer has the greatest sensitivity.
(b) State two advantages of a thermocouple thermometer over a liquid-in-glass
thermometer.
B6 Figure B6.1 shows a pattern of a wave on a string.

48 MOUNT K PUBLISHERS (0979595713)


(a) Name the type of wave shown in figureB6.1.
(b) What is the period of the wave?
(c) How many waves are completed in one second?
(d) How far is point ‘R’ from rest position?
(e) State two similarities between longitudinal and transverse waves.
B7 Figure B7.1 shows an optical pin placed 2cm from a convex lens.

(a) On the figure B7.1 above draw two rays to locate the position of the image.
(b) If the magnification was 3, how far is the image from the lens?
(c) Using the answer in (a) above, calculate the focal length of the lens.
B8 Figure B8.1 shows an electric circuit in a clothes dryer containing two heaters X and Y
connected to a 230V power supply.

49 MOUNT K PUBLISHERS (0979595713)


When both switches are closed, the current in X is 3.5A.
(a) Calculate the power developed in heater X.
(b) The resistance of X is double that Of Y. Determine the total resistance of X and Y.

B9 The element (117


58 ) emits an alpha particle to form element Z.

(i) What is an alpha particle?


(ii) Write the decay equation which represents the above emission.

Section C [20 marks]


Answer any two (2) questions from this section in the Answer Booklet provided.
C1 A train starts from rest at a station and travels with uniform acceleration of 0.5m/s2 for
20 seconds. It then travels with a constant velocity for 30 seconds. Brakes are applied and
the train decelerates uniformly to rest in a further 10 seconds.
(a) Determine the velocity of the train at 20 seconds.
(b) Use the information above to draw to scale, a velocity-time graph of the
described motion on the graph paper provided.
(c) Using the graph drawn; Calculate the:
(i) retardation of the train;
(ii) total distance covered by the train in kilometres.

50 MOUNT K PUBLISHERS (0979595713)


C2 Figure C2.1 below shows a circuit that a Grade J2 learner connected when trying to
measure the resistance of resistor R.

(a) What name is given to the energy supplied by component E?


(b) Identify instruments P and Q.
(c) What is the function of component 5?
(d) The table in figure C2.2 below shows the different current and voltage readings
the pupil obtained by using the setup in figure C2.1.
Current / A 0 0.20 0.30 0.50 0.75 1.00
Voltage / V 0 1.15 1.65 2.65 4.10 5.55
Figure C2.1
(i) On the graph paper provided, draw a graph of voltage against
current. [4]
(ii) Use your graph to find the value of the resistance R. [2]

C3 Strontium — 90 (90
38 ) is a radioactive isotope. A nucleus of strontium — 90 decays by

the emiS5ion of a beta — particle ( )


(a) Define the term isotope
(b) For a neutral atom of strontium — 90, state the
(i) number of electrons
(ii) number of neutrons
(c) When the nucleus of strontium — 90 decays, it becomes a nucleus of Yttrium
(symbol Y). Write the equation for this decay?

51 MOUNT K PUBLISHERS (0979595713)


(d) A sample initially contains 6.0 x 108 atoms of strontium — 90. If the half-life of
strontium — 90 is 29 years, calculate the number of strontium — 90 atoms that
remain in the sample after 87 years?
(e) A teacher says that the emission of beta particles from strontium — 90 is random.
Explain what is meant by ‘random’ in this context.

NOTES!!!
All rights reserved, hence, circulating this booklet in any social media is
illegal.
Appreciate and support the work by donating any amount to the rightful
owner Mr Musonda Laurent - 0974794056 / 0965038377 for betterment of
work.
FOR OTHER SUCCESS BOOKLETS CONTACT MR MUSONDA LAURENT ON THE
FOLLOWINGS:
CALL / TEXT + 260965038377 / +260974794056 / +260954482350
WHATSAPP: +260965038377 OR EMAIL: laurentmsnd@gmail.com

Centre Candidate
Number Number

Candidate Name

EXAMINATION COUNCIL OF ZAMBIA

Examination for School Certificate Ordinary Level

Science 5124/1
Paper 1

Friday 18 NOVEMBER 2018


SECTION A [20 MARKS]
Answer all the questions on the answer grid provided.
A1 Concert 2GB (giga byte) to standard unit in bytes.
A 20B B 200B
C 200 000B D 2000 000 000B
A2 The diagram below shows a micrometer screw gauge.

52 MOUNT K PUBLISHERS (0979595713)


What is the reading shown on the micrometer?
A 5.43mm B 7.30mm C 6.63mm D 8.13mm
A3 The diagram below shows a distance-time graph for a moving object

What physical quantity does the slope of the graph represent?


A Speed B. Velocity C. Displacement D. Acceleration
What happens to the mass and weight of a liquid in a container when moved from earth
to the Moon?
Mass Weight
A Decrease Stay the same
B Decrease Decrease
C Stay the same Stay the same
D Stay the same Decrease

A5 The forward force of a bicycle travelling at a constant speed along a level road is…
A less than the frictional force.
B greater than the frictional force.
C In equilibrium with the fractional force.

53 MOUNT K PUBLISHERS (0979595713)


D acting in the same direction as the frictional force.
A6 The diagram shows a uniform plank in equilibrium when two weights are hung on
opposite sides of the pivot.

What is the size of weight F?


A 50N B 200N
C 300N D 500N

A7 A man pushes a box along an incline by applying a 300N force as shown below.

How much work 1s done by the man in pushing the box?


A 900J B 1200J
C 1500J D 3000J
A8 Which of the following graphs shows the relationship between volume and the kelvin
temperature of a fixed mass of gas at constant pressure?

54 MOUNT K PUBLISHERS (0979595713)


A9 The diagram below shows a water wave.

What is the wavelength of the wave?


A 20cm B 30cm
C 60cm D 80cm

A10 The diagram below shows a ray of light incident on a plane mirror.

If the mirror is rotated clockwise through an angle of 10°, which diagram correctly
Shows the angle through which the reflected ray rotates?

55 MOUNT K PUBLISHERS (0979595713)


A11 What happens to Light as it passes from glass to air? It’s...
A wavelength Increases because its speed Increases.
B frequency decreases because its speed decreases.
C frequency Increases because its speed Increases.
D wavelength decreases because its speed decreases.
A12 A sound wave of frequency 400Hz travels in air at a speed of 340m/s. What is the
distance from the centre of a compression to the centre of the nearest rarefaction in the
waved?
A 0.425m B 0.850m C 1.275m D 1.800m

A13 A bar magnet was broken into three pieces as shown in the diagram below

what will be the polarity of the ends labelled E, F, G and H?


E F G H
A North North South South
B South North South North
C South North North South
D North South North South

56 MOUNT K PUBLISHERS (0979595713)


A14 A polythene rod can be charged negatively by rubbing with a wooIIen cloth. How does
the rod gain its charge? By …
A gaining protons. B losing protons.
C basing electrons. D paining electrons.
A15 The diagram below shows a circuit

What is the reading on voltmeter V2?


A 3V B 6V C 9V D 15V
A16 If the electrical energy costs K2 per unit, what ls the cost of running a 2KW heater for 3 hours?
A K12.00 B K18.00
C K24.00 D K40.00
A17 Why is soft Iron used in the core of the coil in a direct current motor? To...
A present the coil from spinning slower.
B prevent the coil from spinning faster.
C reduce the strength of the magnetic field.
D Increase the strength of the magnetic field.

A18 The diagram below shows a coil in a magnetic field. The coil ls part of a d.c. motor.

What must be connected directly to J and K and in which direction will the coil rotate?
Part connected Direction of rotation

57 MOUNT K PUBLISHERS (0979595713)


A Split rings Clockwise
B Split ring commutator Clockwise
C Split rings Anticlockwise
D Split ring commutator Anticlockwise

A19 The symbol 238


92 represents a nuclide notation for uranium. Which of the following gives
the correct number of electrons and neutrons for the nudide?
Electrons Neutrons
A 92 330
B 92 146
C 146 92
D 330 92

A20 Which of the following equations shows a nuclear fission reaction?


15
A 14
7 + 1
0 → 7

B 46
21 → 46
21 +

C 2
1 + 2
1 → 3
2 + 1
0 +

D 239 → 141 +
92
95 + 31 +

SECTION B [45 MARKS]


Answer all questions in this section.
Write your answers in the spaces provided on the question paper.

B1 Figure B1.1 and B1.2 below show two tank containing liquids M and N.

58 MOUNT K PUBLISHERS (0979595713)


(a) What is the volume of each liquid in m3?
M .............................................. [1]
N: ............................................ [1]
(b) What would be the mass of 1m3 of liquid M?

Mass .............................................................. [2]


(c) Calculate the density of liquid N.
Density: ............................................. [1]
[Total: 5 marks]
B2 Figure B2.1 shows some of the force acting on a car of mass 800kg.

59 MOUNT K PUBLISHERS (0979595713)


(a) State the size of the total drag force when the car is travelling a constant speed.
……………………………………………………………………………………………………………………….. [1]
(b) If the driving force is increased to 3200N
(i) Find the resultant force on the car at this instant.

Resultant force ......................................... [1]


(ii) Calculate the initial acceleration of the car.

Acceleration: .............................................................. [2]


[Total 4 marks]
B3 Figure B3.1 below shows an aeroplane flying at 120 metres above the ground. A box of
mass 1.2kg is seen falling off the aeroplane.

(a) Calculate the weight of the box. (Take g = 10N/kg).


Weight: ........................................................... [2]
(b) What would be the kinetic energy of the box just before hitting the ground?
Kinetic Energy: …………………………………………………. [3]
[Total: 5 marks]

60 MOUNT K PUBLISHERS (0979595713)


B4 Figure B4.1 below shows a pulley which lifts an 8kg wooden block through a vertical
distance of 3 metres by applying an effort of 40N. (Take g = 10N/kg)

(a) Calculate the mechanical advantage of the pulley system above.

M.A ....................................................... [3]


(b) If the pulley wheel above has the velocity ratio (V.R.) = 3.
What is the efficiency of the pulley?

Efficiency… ............................................ [2]


(c) Why is the efficiency calculated in (b) (i) less than 100%?
……………………………………………………………………………………………………………………………
……………………………………………………………………………………………………………………….. [1]
[Total: 6 marks]
B5 Figure B5.1 below shows a dot representing the position in the centre of a rectangular
hall where a learner is standing. The width of the ball is 17m.

When the learner hits a drum two echoes at time intervals of 50 x 10-3 seconds
and 80 x 10-3 seconds respectively are heard by the learner after the bangs.
Assuming that there Is no echo from the ceiling:
61 MOUNT K PUBLISHERS (0979595713)
(a) Calculate the speed of sound in air

Speed of sound: ..................................... [2]


(b) The length of the hall

Length: .................................................. [2]


[Total: 4 marks]
B6 Figure B6.1 below shows a ray of light travelling from water to air.
The Incident ray makes an angle of 65o with the boundary. Most of the light passes into
air and is refracted making an angle of 56o with the boundary.

Figure B6.1
(a) Complete the ray in Figure B6.1 to show the retracted ray. Label the angle of
incidence and angle of refraction.
(b) Calculate the refractive index of water.

Refractive index = ....................... [2]


(c) When the angle of incidence in figure B6.1 is increased, total internal reflection
will occur. State one condition necessary for total internal reflection.
……………………………………………………………………………………………………………………… [1]
[Total: 6 marks]

62 MOUNT K PUBLISHERS (0979595713)


B7 Table B7.1 below stows the melting and boiling points of lead and oxygen.
Melting point (oC) Boiling point (oC)
Lead 327 1744
Oxygen - 219 - 183
Table B7.1
(a) In what physical state of matter will
(i) Lead be at a temperature 450 °C
…………………………………………………………………………………………………………… [1]
(ii) Oxygen be at a temperature of -200°C
………………………………………………………………………………………………………….. [1]
(b) Figure B7.2 below is a diagram of a metallic hot water storage tank showing how
water can be heated using an electric immersion heater bottom.

(i) How could heat loss from the tank be reduced?


…………………………………………………………………………………………………… [1]
(ii) What material would be suitable to be used for the process named in (b)(i)
above?
…………………………………………………………………………………………………… [1]

63 MOUNT K PUBLISHERS (0979595713)


B8 Figure B8.1 below shows a circuit diagram with a lamp connected to a 12V battery.

(a) Calculate the charge passing through the battery in 2 minutes?

Charge............................................. [2]
(b) How much energy is transferred by the battery?

Energy: ......................... [2]


(c) Calculate the resistance of the lamp.

Resistance ............................ [2]


B9 During a radioactive decay, Thorium - 232 (232
90 ℎ) emits two beta particles to form a

new daughter element whose symbol is U.


(a) What is radioactivity?
……………………………………………………………………………………………………………………….. [1]
(b) Construct a decay equation to show how Thorium - 232 decays to U.
……………………………………………………………………………………………………………………… [2]
(c) Determine the number of protons and neutrons in the nucleus of the daughter
element — (U) formed.
Protons: ........................... [1]
Neutrons: ......................... [1]
[Total: 5 marks]

64 MOUNT K PUBLISHERS (0979595713)


Section C [20 marks]
Answer any two (2) questions from this section in the Answer Booklet provided.
C1 A car travels along a straight road. The speedometer readings every 5 seconds are shown
in Table C1.1 below.
Time (s) 0 5 10 15 20 25 30 35 40
Velocity (m/s) 0 10 20 30 30 30 30 15 0
Table C1.1
(a) Draw a velocity-time graph to show the changes of velocity with time. [4]
(b) Using your graph, determine the
(i) distance covered after 20 seconds [2]
(ii) total distance travelled by the car [2]
(iii) average velocity of the car for the whole journey [2]
[Total: 10 marks]
C2 Figure C2.1 below 1s a diagram showing a mercury thermometer placed In warm water.
The reading shown on the thermometer is 50°C.

(a) Define thermal expansion. [1]


(b) (i) What Is the temperature shown on the mercury thermometer in kelvin? [1]
(ii) What property makes it possible for the thermometer to measure
temperature? [1]
(iii) Give any two advantages of using mercury in the thermometer over
alcohol. [2]

65 MOUNT K PUBLISHERS (0979595713)


(c) The difference between the ice point and steam point in a liquid-in-glass
thermometer is 30cm. What temperature is recorded when the mercury thread is
10cm above the ice point? [3]
(d) State one thermometer that can be used to measure high temperatures that vary
considerably. [1]
(e) Name a device that uses thermal expansion to control temperature in a pressing
iron. [1]
[Total: 10 marks]
C3 A Geiger counter was used to measure the activity (in counts per minute) from a
radioactive sample in the laboratory over a period of years. Over this period, the
background radiation was regularly measured at 4 counts/minute.
The table of results ls shown below.
Time in years 0 1 2 3 1 5 6
Recorded activity in count /min 124 80 52 34 23 16 12
Activity due to sample alone 120
Table 3.1
(i) Copy In your answer booklet and complete the table giving the activity of the
sample alone. [1]
(ii) What ls background radiation? [1]
(iii) Give two possible sources of background radiation. [1]
(iv) On a graph paper, plot a graph of the values for the activity of the sample alone
against time. [1]
(v) From the graph, determine the hall-life of the substance. [1]

66 MOUNT K PUBLISHERS (0979595713)


Centre Candidate
Number Number

Candidate Name

EXAMINATION COUNCIL OF ZAMBIA

Examination for General Certificate of Education Ordinary Level

Science 5124/1
Paper 1

Friday 2 August 2019

Additional information:
Electronic calculator (non-programmable) and / or Mathematical tables
Graph papers
Soft clean eraser
Soft pencil (type B or HB is recommended)

Time 2 hours

SECTION A [20 MARKS]


Answer all the questions on the answer grid provided.
1. The diagram below shows 4 identical metal bars placed between two wooden blocks on a
ruler.

What is the diameter of one metal bar?


A. 1.0 cm B. 2.0 cm C. 3.0 cm D. 4.0 cm

2. Which of the following would change the period of oscillation of a pendulum?


A. Thickness of the string C. Mass of the pendulum of bob
B. Length of the pendulum D. Volume of the pendulum bob

67 MOUNT K PUBLISHERS (0979595713)


3. The diagram below is a graph showing the movement of a car over a period of 70
seconds.

What distance was travelled by the car while its speed was decreasing?
A. 100 m B. 200 m C. 400 m D. 500 m
4. The diagram below shows a 200g object on a frictionless surface acted upon by 3 forces.

Which of the following is correct about the acceleration and direction of the movement
of the object?
Acceleration (m/s2) Direction of movement
A 0.05 Left
B 0.05 Right
C 5.0 Right
D 500 Left

5. Why would a mechanic prefer to use a longer spanner to loosen a nut than a short one?
A longer spanner would ………………………
A. Add grease to the nut.
B. Allow a mechanic to apply more force.
C. Give a greater turning effect
D. take a longer time to loosen the nut

68 MOUNT K PUBLISHERS (0979595713)


6. A drum of water of weight 2000N is rolled up a plank of length 2m unto a platform of
0.8m high.

What is the work done on the drum against gravity?


A. 1000J B. 1600J C. 2500J D. 4000J
7. A crane lifts a 600kg mass through a vertical height of 12m in 18 seconds. What is the
crane’s power output?
A. 4 000W B. 6 000W C. 10 000W D. 72 000W
8. The diagram below shows a wheel and axle (not drawn to scale) used to raise a load of
280N by a force of 40N.

If the radii of the wheel ® and axle (r) are 70cm and 5 cm respectively. What is the
velocity ratio of the wheel and axle?
A. 0.07 B. 7 C. 14 D. 50

69 MOUNT K PUBLISHERS (0979595713)


9. The diagrams below show a bimetallic strip before and when it was dipped in a liquid of
temperature – 20 oC.

Which of the following is correct?


A. Metal A is a poor conductor of heat than metal B.
B. Metal B is a better conductor of heat than metal A
C. Metal A contrasts more than metal B when cooled.
D. When cooled metal B contrast more than metal A.
10. A bicycle pump contains 70cm3 of air at a pressure of 1.0 atmosphere and temperature
of 7 OC. what is the pressure when the air is compressed to 30 cm3 at temperature of 27
o
C?
A. 0.4 atmospheres B. 2.5 atmospheres
C. 280 atmospheres D. 300 atmospheres
11. The diagram below represents positions at one particular time on a longitudinal wave.

Which positions are one wavelength apart?


A. W and X B. W and Z
C. X and Z D. Y and Z
12. Which of the following of sound frequencies has the highest pitch?
A. 150 Hz B. 200Hz
C. 400Hz D. 500Hz

70 MOUNT K PUBLISHERS (0979595713)


13. The diagram below shows a ray of light from air into a rectangular glass block.

1 2
= =
3 3

3 4
= =
1 2

14. The diagram below shows how an image is formed from an object placed in front of a
converging lens.

What is the magnification?


A. 0.2 B. 1.8 C. 2.0 D. 4.5
15. A negatively charged rod is held close to but not touching an insulated metal sphere.

Which diagram below shows the charges that are induced on the sphere?

71 MOUNT K PUBLISHERS (0979595713)


16. A small heater operates at 12V, 2A. How much energy will it use when it is switched on
for 5 minutes?
30 J B. 120 J C. 1800 J D. 7200 J
17. The diagram below show magnetic field patterns between North and South poles of
magnets.

Which of numbered diagram shows the correct magnetic field pattern?


A. 1 B. 2 C. 3 D. 4
18. A transformer has 1000 turns on the primary coil. The voltage applied to the primary coil
is 230V a.c. how many turns are on the secondary coil if the output voltage is 46V?
A. 20 B. 200 C. 400 D. 800
19. What is the purpose of X – plates in a cathode ray oscilloscope? To…
A. Deflect the electron beam vertically.
B. Make the waveform brighter on the screen.
C. Deflect the electron beam horizontally
D. Produce a waveform across the screen.

72 MOUNT K PUBLISHERS (0979595713)


20. A sodium nucleus decays by emission of Beta particle to form magnesium. Which
equation is the correct representation of the decay?
B. 24
11 − −→ 20
9 + 4
2

24 0
C. 24
11 − −→ 12 + −1

D. 24
11 − −→ 24
11 +

E. 24
11 − −→ 28
13 + 4
2

Section B [45 marks]


Answer all questions in this section
Write your answers in the spaces provide on the question paper

B1 The diagram below shows a displacement – time graph for the motion of a dog over a
period of 12s.

73 MOUNT K PUBLISHERS (0979595713)


(a) Use the graph to calculate
(i) The velocity of the dog at 10s.

Velocity ................................ [2]


(ii) The acceleration of the dog from 0 to 10 seconds.

Acceleration ................................. [2]


(b) From the graph, determine the time when the dog is not moving.
…………………………………………………………………..………………………………………………………[1]

B2 The diagram below shows a uniform plank of length 100cm kept in equilibrium by a 3kg
and 1kg mass placed 10cm and 20cm from each end respectively.

(a) Calculate the


(i) moment of the 3kg mass [2]
(ii) weight of the plank [2]
(b) State one application of the above set up. [1]
………………………………………………………….…………………………………………………………………

74 MOUNT K PUBLISHERS (0979595713)


B3 The diagram below shows a spring catapult design by a pupil for a science project. The
catapult consists of movable plunger with a spring attached to it.

A metal ball of mass 0.2kg was placed on the metal plate and the handle of the catapult
pulled to fully compress the spring. On release of the handle, the ball was projected 1.5m
vertically.
(a) Name the type of energy stored in a compressed spring.
…………………………………………………………………………………………………………….. [1]
(b) Calculate the maximum potential energy acquired by the ball from the catapult.

Energy ........................................... [2]


(c) Give one reason why the potential energy you have calculated in (b) is less than
the original stored energy of the spring.
…………………………………………………………………………………………………………………………. [2]

75 MOUNT K PUBLISHERS (0979595713)


B4 The diagram below shows a block and tackle pulley system used to lift a load of 200kg.

(a) State the velocity ratio of the pulley system in the diagram above.

Velocity ratio ............................. [1]


(b) If the machine has an efficiency of 80%, calculate the effort applied.
(Take g = 10N/kg)

Effort ....................................... [3]

76 MOUNT K PUBLISHERS (0979595713)


B5 Figure B5.1 is a graph of how a wave is propagated.

(a) From the graph determine the


(i) Amplitude of the wave.

……………………………………………………….. [1]
(ii) Wavelength of the wave

………………………………………………………. [1]
(b) Calculate the frequency of the wave if it travels at 3m/s.

Frequency .................................. [3]

77 MOUNT K PUBLISHERS (0979595713)


B6 Figure B6.1 shows a Perspex disc fitted on a wooden stand.

When rubbed with fur, electrons move from the Perspex disc to the fur.
(a) (i) what charge is induced in the Perspex disc?
………………………………………………………….. [1]
(ii) State the unit in which charge is measured.
…………………………………………………………. [1]
(b) Small pieces of paper brought near the Perspex disc stick to it.
Explain why.
………………………………………………………………………………………………………………………[1]
(c) When a polyethene rod was rubbed with fur, it was able to attract small pieces of
paper too. Name the type of charge induced in the polythene rod and explain
using movement of electrons how charge was induced.
Name of charge ................................................ [1]
Explanation: ………………………………………………………….
……………………………………………………………………….[1]

78 MOUNT K PUBLISHERS (0979595713)


B7 Figure B7.1 shows a circuit diagram connected to a 12V supply.

Calculate the
(a) Combined resistance of the 2Ω and 3Ω resistors.
(b) Total resistance in the circuit when the switch is closed.
(c) Potential difference across the 2Ω resistor.
(d) Total current flowing in the circuit when the switch is closed.

B8 The diagram below shows an iron core electromagnet.

(a) State two ways in which the strength of the electromagnet can be increased.
1............................................................................. [1]
2 ……………………………………………………………. [1]
(b) Explain why the core of the electromagnet is made of iron instead of steel.
…………………………………………………………………………………………………………………………[2]
79 MOUNT K PUBLISHERS (0979595713)
B9 Figure 9.1 shows a decay curve of a radon nuclide.

(a) Define nuclear fission.


…………………………………………………………………………..
(b) From the graph, determine the
(i) Original mass of nuclide.

(ii) Quantity of the nuclide that remained after 6 days.

(iii) Half-life of the nuclide.

80 MOUNT K PUBLISHERS (0979595713)


Section C [20 marks]
Answer any two (2) questions from this section in the Answer Booklet provided.
C1 The table below shows the thinking distance for various speeds for a driver to react and
apply brakes before stopping.
Thinking distance (m) 0 9 12 15
Speed (km/h) 0 48 64 80

(a) (i) On a graph paper, draw a graph of thinking distance against speed. [4]
(ii) Use your graph to determine the thinking distance for the driver at 32km/hr. [2]
(b) The driver then drinks two bottles of alcohol. After sometime his thinking time
was measured as 1.0 second.
(i) Calculate the thinking distance for the driver when driving at 32km/hr. [3]
(ii) What was the effect of the alcohol on the thinking distance of the driver?

C2 (a) The table below shows the results obtained in an experiment to


determine refractive index of paraffin using real and apparent depth.
Real depth (cm) 4.0 6.0 8.0 10.0 12.0 14.0
Apparent depth (cm) 2.8 4.2 5.6 7.0 8.4 9.8

(i) Draw a graph of real depth against apparent depth using the values in
table above. [4]
(ii) What does the gradient of the line in the graph you plotted in (a)(i) above
represent. [1]
(iii) Using the graph determine the refractive index of paraffin. [2]
(b) The refractive index of oil is 1.45 and that of water is 1.33.
What angle does a ray of light incident to oil at 40O make with water when oil
floating on water. [3]

81 MOUNT K PUBLISHERS (0979595713)


C3 (a) Sodium chloride salt containing sodium -24 is radioactive and is used
in medicine. The sodium in the salt decays according to the equation.

24
11 − −> 24
12 + + −

(i) What is particle W? [1]

(ii) How many neutrons are in the nucleus of sodium – 24? [1]

(iii)
Explain the difference between an atom of sodium – 24 and atom of
sodium – 23. [2]
(b) Gamma radiation is used to treat food so that it stays fresh. To check whether the
treated food is safe for consumption, the count rates of three food samples X, Y and Z
were measured before and after treatment with gamma rays. Table C3.1 shows the
results obtained.
Food Count rate before Count rate after treatment /
sample treatment / counts per count per minute
minute
X 30 33
Y 31 27
Z 30 31

(i) Explain why there is a measured count rate before the food sample are treated
with gamma rays. [1]
(ii) From the results, determine whether the food sample Y becomes radioactive or not.
(iii) State one other use of gamma radiation. [1]
(iv) State two safety precautions that must be followed when handling gamma
radiation. [2]

82 MOUNT K PUBLISHERS (0979595713)


Centre Candidate
Number Number

Candidate Name

EXAMINATION COUNCIL OF ZAMBIA

Examination for School Certificate Ordinary Level

Science 5124/1
Paper 1

Friday 18 NOVEMBER 2019

Time 2 hours

SECTION A [20 MARKS]


Answer all the questions on the answer grid provided.
A1. Which of the following instruments would be used to accurately measure the inside
diameter of a bottle neck?
A. Ruler B. Vernier caliper
C. Engineer’s caliper D. Micrometer screw gauge
A2. The diagram below shows a bob of a simple pendulum that swings from position A to B.

If it takes the bob 0.5 seconds to swings from A to B, what is the period of the pendulum?
A. 2.0s B. 3.0s C. 4.0s D. 5.0s

83 MOUNT K PUBLISHERS (0979595713)


A3. The figure below shows a graph of how the speed of 2 athletes changed with time.

Which of the followings is true about the athletes at time t?


A. Athlete 2 is slowing down
B. Athlete 1 is overtaking athlete 2
C. Athlete 1 has zero acceleration
D. Both athletes are moving at the same speed.

A4. What is the weight of a 75 000g man on the moon where the gravitational field strength
is one – sixth of 10N/Kg?
A. 75 N B. 125N C. 12 500N D. 75 000N
A5. Why is a bus carrying luggage on the upper carrier more unstable than the one carrying
luggage on lower carrier? This is because the…..
A. Centre of gravity is raised
C. weight of the bus increase
B. Friction on the tyres increases
D. pressure on the tyres increases

84 MOUNT K PUBLISHERS (0979595713)


A6. A ball of mass 0.50kg is released from a height of 1.0metre and falls towards the floor.

Which row in the table below shows the potential energy and kinetic energy of the ball
when it is at a height of 0.25metres from the floor?
P.E (J) K.E (J)
A 0.12 0.12
B 1.25 1.25
C 1.25 3.75
D 3.75 1.25

A7. An elevator carries 10 people of average mass 70kg each to a height of 5 metres in 2
minutes. What is the power generated by the elevator?
A. 29.17W B. 291.67W
C. 1 750W D. 1 7500W
A8. The figure below is a diagram showing how a pair of scissors is used to cut a piece of cloth.

Which letters below correctly identifies parts labelled X, Y and Z?

85 MOUNT K PUBLISHERS (0979595713)


Pivot Effort Load
A X Y Z
B Z Y X
C Y X Z
D Y Z X

A9. The diagram below shows the variation of temperature with time as a solid substance
was heated.

Which of the following statements is true about the graph above? In region …
A QR – condensation was taking place.
B ST –the substance was melting.
C RS – the substance in liquid state.
D TU – the substance was in solid state.
A10. What happens to the volume of a fixed mass of a gas when it is heated at constant
pressure? The volume of a gas ..
A Reduced B. increases
C. is halved D. remained the same

86 MOUNT K PUBLISHERS (0979595713)


A11. The figure below is a diagram of a water wave that shows a small cork Z moving up and
down through one complete oscillation every 4 seconds.

What is the speed of the wave?


A 0.25m/s B. 0.75m/s
C. 4.00m/s D. 12.00m/s
A12. The diagram below shows a ray of light incident on the edge of a piece of glass.

If the angle of incidence, i, is greater than the critical angle, which numbered ray shows the
direction of light after it leaves the edge of the glass.
A. 1 B. 2 C. 3 D. 4
A13. A ray of light passes through a semi – circular glass block as shown below.

87 MOUNT K PUBLISHERS (0979595713)


What is the refractive index of air?
A 0.56 B. 0.97 C. 1.10 D. 1.91
A14. A person sees a flash of lightening and hears the thunder 4.8s later. If the speed of sound
in air was 330m/s, how far was the lightening from the person?
34.38m B. 68.75m C. 1 584m D. 3 168m
A15.Which of the following is a property of a magnet?
A. Attract all metals brought near it.
B. Repels all magnets brought near it.
C. When freely suspended it lies in the east – west direction.
D. When freely suspended it lies in the north – south direction.
A16. Which of the following will increase the resistance of a metal wire in a circuit?
Size of a wire Temperature of wire
A Longer Lower
B Thinner Higher
C Shorter Higher
D Thinner Lower

A17. The diagram below shows two resistors in series connected to a 3V battery.

The ammeter has negligible resistance. What is the resistance of R?


A 0.5 Ω
B 1.5 Ω
C 5Ω
D 6Ω

A18. The graph below shows a trace on the screen of an oscilloscope when connected to a
transformer.

88 MOUNT K PUBLISHERS (0979595713)


If the Y-control is set to 5V per centimetre, what is the value of the peak voltage?
A 4V B 5V
C 10V D 20V
A19. Which one of the following travels in a straight line across a magnetic field?
A Proton B Gamma ray
C Beta particle D Alpha particle
A20. 90 ℎ) with a half – life of 24 days decays by emitting a
The radioactive element thorium (234
1
beta particle. How long will it take for 1g of thorium to decay to of the mass?
8

A 72 days B 96 days
C 120 days D 192 days

89 MOUNT K PUBLISHERS (0979595713)


Section B [45 marks]
Answer all questions in this section
Write your answers in the spaces provide on the question paper
B1 The diagram below shows a graph showing how the velocity of a car changed when the
driver applied brakes upon seeing a herd of cattle crossing the road ahead.

Use the information on the graph to calculate the


(a) Deceleration of the car between point A and B. [2]
(b) Distance moved by the car in the 10 seconds. [2]
(c) Average speed of a car. [2]

90 MOUNT K PUBLISHERS (0979595713)


B2 The diagram below shows a device designed to compress a crushed material for school
science project.

(a) State one condition necessary for a body to maintain equilibrium. [1]

(b) Calculate the moment of the 50N force about the hinge. [2]

(c) Calculate the upward force F, which the crushed material exerts on the piston to keep
the beam in equilibrium. [2]

B3 The diagram below shows some energy conversions in a coal powered station.

(a) How much useful electrical energy is provided by the power station? [1]

(b) Calculate how much energy is wasted from the cooling tower. [2]

(c) Calculate the efficiency of the power station in the production of electrical energy.

91 MOUNT K PUBLISHERS (0979595713)


B4 Figure B4.1 shows a cylinder containing a gas ogf volume 5cm3 at a pressure of 100kpa.
The piston is pushed to half the original volume at the same temperature as shown in
figure B4.2.

(a) state boyle’s law [1]


(b) Calculate the gas pressure in the cylinder in figure B4.2. [2]
(c) Use the kinetic theory to explain why pressure increases in the cylinder as volume
decreases. [2]
B5 (a) The human audible frequency range is 20Hz to 20 000Hz. Taking the
speed of sound in air as 330m/s.
(i) What is the upper limit of the human audible frequency in kHz?
(ii) What is the wavelength of the lower limit of the audible frequency
range? [2]
(b) The diagram below shows a wave travelling through air.

Which labelled distance represents


(i) One wavelength,
(ii) The amplitude of the wave

92 MOUNT K PUBLISHERS (0979595713)


B6 The diagram below shows the appearance of a coin immersed in water when viewed
from above.

(a) State Snell’s law [1]


(b) What happens to the rays of light at the surface of water in the diagram above? [1]
(c) Calculate the refractive index of water. [2]

B7 Figure B7.1 is a circuit diagram with two resistors connected to a 3V power supply.

(a) Identify the type of circuit arrangement of the resistors shown in the circuit
above. [1]
(b) Calculate the total resistance in the circuit. [1]
(c) Calculate the ammeter reading when the switch is closed. [2]
B8 A cell phone charger has a transformer with 2 000 turns in the primary coil which converts
240V a.c to 6V a.c and a rectifier that converts 6V a.c . to 6V d.c.
(a) What do the letters a.c. and d.c. stand for?
(i) a.c. …………………………………………………...
(ii) d.c. ………………………………………………….. [2]

93 MOUNT K PUBLISHERS (0979595713)


(b) How many turns are in the secondary coil of the transformer? [2]
(c) Draw a labelled diagram of a step down transformer. [2]
B9 The symbol 23892 represents a nucleus of uranium with nucleon number 238 and proton
number 92.
(a) State the meaning of nucleon number. [1]
(b) A nucleus of uranium-238 decays to form a nucleus of thorium by the emission of
an alpha particle. State the
(i) proton number of an alpha-particle, [1]
(ii) nucleon number of an alpha-particle, [1]
(iii) proton number of thorium, [1]
(iv) nucleon number of thorium isotope. [1]
Section C [20 marks]
Answer any two (2) questions from this section in the Answer Booklet provided.
C1 A spring with its upper end and fixed, hangs vertically alongside a metre rule. The lower
end of the spring gave the following readings in table when various masses were hung
from it.
Mass (kg) 0 0.02 0.04 0.06 0.08 1.00
Reading (cm) 10.0 12.0 14.0 16.0 17.0 17.5
Applied force (N) 0 1.0
Extension (cm) 0 7.5

(a) State Hooke’s law [1]

(b) Copy and complete the table for the values of applied force (N) and extension
(cm) produced. [4]

(c) Draw a graph pf applied force against extension on a graph paper. [4]

(d) On the graph, show the elastic limit. [1]

[ Total: 10 marks]

94 MOUNT K PUBLISHERS (0979595713)


C2 Two lamps S and T of resistance 200Q and 3000 respectively are connected in series with
a 240V mains as shown in figure C2.1 below.

(a) Calculate the


(i) current at point V, [2]
(ii) potential difference across lamp S. [1]
(b) Draw a circuit diagram showing resistors S and T connected in parallel. [1]
(c) Using the diagram in 2 (b), calculate the
(i) current through lamp S and lamp T,
(ii) total resistance of the circuit. [4]
(d) Give two reasons why lamps are preferably connected in parallel in homes. [2]
C3 The diagram below shows a bowling ball as it rolls down in incline for 5.0 seconds before
reaching the horizontal ground.

The changes in gravitational potential energy of the bowling ball as it rolls down to the
horizontal ground are shown in the table below.
Time elapsed in seconds 0 1 2 3 4 5
Potential energy in joules 9 7.2 5.4 3.6 1.8 0
(a) State the law of conservation of energy. [1]
(b) Using the values in the table, draw a graph of a potential energy against time. [4]
(c) Calculate the height, h of the inclined plane, when the potential energy was 9 J, if
the mass of the ball was 900g.
(d) Using the graph, estimate the kinetic energy of the bowling ball at time
(i) 2.5 seconds
(ii) 4.0 seconds

95 MOUNT K PUBLISHERS (0979595713)


Centre Candidate
Number Number

Candidate Name

EXAMINATION COUNCIL OF ZAMBIA

Examination for General Certificate of Education Ordinary Level

Science 5124/1
Paper 1

2020

Additional information:
Electronic calculator (non-programmable) and / or Mathematical tables
Graph papers
Soft clean eraser
Soft pencil (type B or HB is recommended)

Time 2 hours

SECTION A [20 MARKS]


Answer all the questions on the answer grid provided.
Al How many significant figures are there in the number 0.0502007?
A 2 B 3
C 5 D 6
A2 A stone of mass 400g is lowered into a measuring cylinder containing water.
The water level rises from 300cm3 to 500cm3. What is the density of the stone?
A 0.50g/cm3
B 0.80g/cm3
C 1.33g/cm3
D 2.00g/cm3

96 MOUNT K PUBLISHERS (0979595713)


A3 The graph below shows the speed of an athlete during a race.

What is the distance travelled by the athlete?


A 50m B 65m
C 75m D 90m
A4 A 4kg brick is dropped from the top of a building whose height is 30m. What is the
velocity with which it reaches the ground?
A 7.5m/s B 24.5m/s
C 120.0m/s D 1 200.0m/s
A5 The diagram below shows an object moving with a constant velocity when a force E of
30N is applied.

What is the value of the opposing force F?


A ON B 15N
C 30N D 300N

97 MOUNT K PUBLISHERS (0979595713)


A6 A 60N object is placed on a uniform bar that was balanced at its mid-point as shown in
the diagram below.

Which of the following will rebalance the beam?


Magnitude of force Position of force
A 30N 60cm to the left of the pivot
B 30N 60cm to the right of the pivot
C 45N 45cm to the right of the pivot
D 90N 20cm to the left of the pivot
A7 A motor is used to pull a 10kg box along a 5 meter long incline to the top of the decline as
shown in the diagram below.

Ignoring all frictional forces, determine the work done against gravity in pulling the box
from the foot of the incline to the top of the decline.
(Take g as 10N/kg)
A 40J B 50J
C 400J D 500J

98 MOUNT K PUBLISHERS (0979595713)


A8 A girl whose mass is 50kg climbs a ladder of height 8m in 10 seconds. What is the power
developed by the girl?
A 40W B 400W
C 500W D 5 000W
A9 A machine with a velocity ratio of 6 requires 800J of work to raise a load of 60kg through
a vertical distance of 1m. Find the efficiency of the machine.
A 0.45 B 0.75
C 4.5 D 75.0

A10 The diagrams below show the length of mercury threads at the ice and steam points.

What will be the temperature if the length o' the mercury thread is 25mm?
A 5 oC B 8 oC
C 33 oC D 50 oC
A11 All The diagram below shows a cylinder with a gas trapped inside at a constant
temperature. The length of the gas column is 50cm and the pressure of the gas is 20Pa.

99 MOUNT K PUBLISHERS (0979595713)


If the piston is pushed a distance of 30cm inside, what is the new pressure of the gas?
A 8Pa
B 12Pa
C 30Pa
D 50Pa

A12 Which of the following correctly gives the properties of a sound wave?
Nature Speed in air
A Longitudinal 340m/s
B Longitudinal 3.0 x 108m/s
C Transverse 340m/s
D Transverse 3.0 x 108m/s
A13 The diagram below shows the movement of a ray of light from air to plastic.

Which ratio is the refractive index of air?

100 MOUNT K PUBLISHERS (0979595713)


45 45
60 30
30 60
45 45
A14 The diagram below shows the main sections of the electromagnetic spectrum in order of
increasing frequency.

Which of the following is an application used by Q?


A Sterilisation
B Satellite television
C Killing cancerous cells
D Television remote controller

A15 The diagram below shows a coil in a magnetic field, connected to a DC supply.

As the current I flows in the coil, in which direction will the coil rotate and what must be
connected to parts R and T?
Direction of rotation of coil Direction of rotation of coil
A Clockwise Clockwise
B Clockwise Clockwise
C Anticlockwise Anticlockwise
D Anticlockwise Anticlockwise

101 MOUNT K PUBLISHERS (0979595713)


A16 The diagram below illustrates what happens when metal balls H, J and K suspended by
insulating threads are brought closer to each other.

If the charge on H is positive, what charges are on bal! J arc


J K
A Negative Positive
B Negative Negative
C Positive Negative
D Negative Positive
A17 Two resistors are connected in a circuit as shown in the diagram below.

Which labelled ammeter has the highest current reading?


A W B X
C Y D Z
A18 What is the cost of running a 100W lamp for 30 minutes if electrical energy costs K10 per
unit?
A K0.05 B K0.50
C K30.0 D K300.0

102 MOUNT K PUBLISHERS (0979595713)


A19 The following can be used in detecting alpha particles, beta particles and gamma rays.
1 Photographic film
2 Diffusion cloud chamber
3 G-m tube
4 Spark-counter
Which of the above cannot be used to detect beta particles and gamma rays?
A 4 B 1 and 4
C 2 and 4 D 1, 2 and 3
A20 A radioactive source which has a half-life of 1 hour gives a count rate of 100 counts per
second at the start of an experiment and 25 counts per second at the end. How long in
hours did the experiment take?
A 1
B 2
C 3
D 4

Section B [45 marks]


Answer all questions in this section.
Write your answers in the spaces provided on the question paper.
B1 An object of mass 75kg on earth is taken to the moon where the gravitational field
strength is 1.6N/kg,

(a) What is the mass of me object on the moon? [1]

(b) Calculate the weight of the object on the moon. [2]

(c) Give two differences between mass and weight, [2]


[Total: 5 marks]

103 MOUNT K PUBLISHERS (0979595713)


B2 A bullet of mass 50g moving with an initial speed of 500m/s penetrates a wall and comes
to rest in 0.2 seconds.
(a) Calculate the deceleration of the bullet over the 0.2 seconds. [2]
(b) Determine the retarding force acting on the bullet. [2]
[Total: 4 marks]
B3 Figure B3.1 below shows a pulley systems.

(a) State the velocity ratio of the pulley shown in figure B3.1 above. [1]
(b) If the efficiency of the pulley is 75%, what is its M.A? [2]
(c) If the load is 120N, what is the size of the effort? [2]
(d) What work is done by the effort if the load is lifted 0.5m? [2]
B4 Waves travel at a speed of 30m/s through a medium. If 10 waves pass through a point
per second,
(a) calculate the
(i) frequency of the wave, [2]
(ii) wavelength of the wave. [2]
(b) What does loudness of a sound depend on? [1]

104 MOUNT K PUBLISHERS (0979595713)


B5 Figure B5.1 below shows a ray of light moving from one medium to another.

(a) Find the angle of refraction [1]


(b) Calculate the refractive index of medium B. [2]
(c) Which of the two media is denser than the other? [1]
B6 Figure B6.1 shows an object 0 placed in front of a thin converging lens of focal point F.

(a) Complete the ray diagram to locate the position of the image formed by the
converging lens. [2]
(b) State the characteristics of the image formed. [1]
(c) Calculate the magnification of the lens. [2]
[Total: 5 marks]

105 MOUNT K PUBLISHERS (0979595713)


B7 A step-up transformer increases the voltage of an a.c. supply from 110V to 220V. The
primary coil dissipates a power of 1.1 kW and the transformer has an efficiency of 100%.
(a) If the number of turns in the primary coil is 400, how many turns are in the
secondary coil? [2]
(b) How much current flows in the
(i) primary coil, [2]
(ii) secondary coil? [2]
[Total: 6 marks]
B8 Figure B8.1 below shows two pairs of 3.0.0 resistors connected to a 6.0V battery.

Calculate the
(a) resistance in the circuit between M and N, [2]
(b) current through the battery, [2]
(c) power developed in the battery. [2]
[Total: 6 marks]
B9 Radioactive iodine is used to treat tumours of the thyroid gland. It decays by emitting
beta particles and gamma radiation.
The beta emitting process is represented by the following equation:
131 A
53I ZXe + −01e
(a) What is the nucleon number of the new nucleus Xe formed? [1]
(b) The half-life of iodine-131 is 8 days. The total dose of iodine given to a patient
initially emits 4 x 108 rays per second.
How many gamma rays does the total dose of iodine emit each second after 24
days? [2]

106 MOUNT K PUBLISHERS (0979595713)


Section C [20 marks]
Answer any two (2) questions from this section. Write your answers in the separate
Answer Booklet provided.

C1 (a) (i) Explain the term absolute zero temperature. [1]


(ii) State Charles' law. [1]
(b) The table C1.1 below displays results of temperature against volume of air at a
constant pressure of 720mm Hg.
Temp (°C) 17 28 39 60 80 100
Volume (cm3) 7.5 7.9 8.1 8.7 9.1 9.6
Temp (K)

(i) Copy and complete the table above. [2]


(ii) Plot a graph of volume (cm3) against temperature (K). [4]
(iii) From the graph, determine the volume of the air when the temperature
was 77°C. [2]
[Total: 10 marks]

107 MOUNT K PUBLISHERS (0979595713)


C2 Figure C2.1 below shows how a solar heater on a house roof is used to warm up water
for a house.

(a) Why is the panel in the solar heater black? [1]


(b) Why is there an insulating layer behind the panel? [1]
(c) How does the water in the tank get heated? [2]
(d) On average, each square metre of the solar panel receives 1 000 Joules of energy
from the sun every second. Using this information, calculate the power input in
kilowatts of the panel if its surface area is 2m2. [3]
(e) The solar heater in the diagram has an efficiency of 60% (it wastes 40% of the
solar energy it receives). What area of panel would be needed to deliver heat at
the same rate, on average, as a 3kW immersion heater? [3]
[Total: 10 marks]

108 MOUNT K PUBLISHERS (0979595713)


C3 A radioactive substance B has a half-life of 4 years and undergoes radioactivity by giving
out beta (p) radiation.
(a) Which of the containers aluminium, thin plastic or lead, lined would you use to
safely store substance B? [1]
(b) Copy and complete the table C3.1

Date Mass of original radioactive substance left


1 July 2008 8kg
1 July 2012
1 July 2020

Table C3.1
(c) A Geiger counter was used to measure the activity (in counts per minute) from a
radioactive sample in the laboratory over a period of years. Over this period, the
background radiation was regularly measured at 4 counts/minute.
Table C3.2 shows the results.
Time in years 0 1 2 3 4 5 6
Recorded activities in counts/min 124 80 52 34 23 16 12
Activity due to sample alone 120 8
Table C3.2
(i) Copy and complete the table C3.2 on the activity of the sample alone. [1]
(ii) Explain what is meant by background radiation. [1]
(iii) Plot a graph of the values for activity due to the sample alone against the
time. [4]
(iv) Using your graph determine the half-life of the substance. [1]

109 MOUNT K PUBLISHERS (0979595713)


Centre Candidate
Number Number

Candidate Name

EXAMINATION COUNCIL OF ZAMBIA

Examination for School Certificate Ordinary Level

Science 5124/1
Paper 1

2020
Time 2 hours Marks: 85

SECTION A [20 MARKS]


Answer all the questions on the answer grid provided.
Al How many significant figures are in 2.0800?
A 2
B 3
C 4
D 5
A2 Which of the following contains base physical quantities only?
A Length in kilometres, mass in tonnes
B Length in metres, mass in tonnes
C Length in kilometres, mass in kilograms
D Length in metres, mass in kilograms

110 MOUNT K PUBLISHERS (0979595713)


A3 The following diagrams show steps that a learner carried out in order to determine the
volume of a wooden cork.

Which one is the volume of the wooden cork?


A 30cm3
B 40cm3
C 50cm3
D 100cm3

A4 Which of the following diagrams show an accelerating car?

111 MOUNT K PUBLISHERS (0979595713)


A5 The diagram below shows a light bar PR pivoted at P, balanced by a 4N weight and force F.

What is the reading on the Newton balance?


A 2.0N B 2.4N
C 3.0N D 6.0N
A6 A car is travelling on a level road. Suddenly the driver of the car sees a cow crossing the
road and decides to apply brakes. What are the energy changes as the car slows down?
A Chemical to kinetic and sound
B Heat to kinetic and sound
C Kinetic to chemical and sound
D Kinetic to heat and sound
A7 The graph below shows a cooling curve of a gaseous substance.

In which state(s) of matter will the substance be between point F and G?


A Gas only B Liquid only
C Gas and liquid D Solid and liquid

112 MOUNT K PUBLISHERS (0979595713)


A8 The following diagram shows a water wave.

Which row gives the correct values for the wave amplitude and wavelength?
Amplitude/cm Wavelength/cm
A 4.0 10
B 4.0 5.0
C 8.0 5.0
D 8.0 10

A9 The diagram below shows a learner in front of a loudspeaker that produces sound.

Which of the following diagrams best shows how the gaseous air molecules between the
learner and the loudspeaker will move?

113 MOUNT K PUBLISHERS (0979595713)


A10 A girl of height 1.2 metres stands 4.0 metres in front of a plane mirror as shown in the
following diagram:

How far is the girl away from her image?


A 1.2m B 2.4m C 4.0m D 8.0m

A11 A line 3K drawn on a piece of plain paper on which a glass block is placed was viewed
through the top and its image was located using a pin as shown in the following diagram.

If KL = 1.8cm and KM = 4.8cm, what is the refractive index of the glass?


A 0.63 B 1.50
C 1.60 D 2.66
A12 Which of the following materials would be most suitable for constructing the core of an
electromagnet?
A Carbon B Copper
C Iron D Steel

114 MOUNT K PUBLISHERS (0979595713)


A13 Two uncharged metal spheres Q and P are placed on insulating stands and are separated
while the negatively charged polythene strip is held near Q as shown in the diagram.

What would be the charges on Q and P?


Q P
A + +
B + -
C - +
D - -

A14 The following diagram shows a network of 3 resistors R1, R2 and R3 connected to a
battery.
Voltmeters V1, V2 and V3 are connected to R1, R2 and R3 respectively.

Which of the following expressions show the potential difference (p.d.) supplied by the
battery?
A V1 + V3
B V2 + V3
C V1 + V2 + V3
D V1 - (V2 + V3)

115 MOUNT K PUBLISHERS (0979595713)


A15 An electric kettle is rated '230V, 3 000W'. What is the suitable fuse to use on this kettle?
A 2A B 5A
C 10A D 15A
A16 Below are 3 statements on electromagnetism.
1. An electromagnet consists of a coil of wire wound on a soft iron core.
2. The strength of the magnetic field produced by an electromagnet increases if the
strength of the current and/or number of turns of wire is increased.
3. In the diagram below, when the switch is closed, the gap, R, increases.

Which of the above statements are true?


A 1 and 2 B 2 and 3
C 1 and 3 D 1, 2 and 3
A17 The diagram below shows a filament lamp rated 6.0V connected to the output of a
transformer.

What happens to the lamp when the circuit is switched on?


A Lights dimly B Does not light at all
C Lights at normal brightness D Lights up brightly and then blows off

116 MOUNT K PUBLISHERS (0979595713)


A18 Which component used in electronic circuits allows current to flow through in one
direction only?
A Diode B Resistor
C Thermistor D Transformer
A19 An atom of uranium-235 has 92 electrons. How many protons are there in one atom of
uranium-235?
A 92 B 143
C 235 D 327
A20 A radioactive material gives a count rate of 8 000 counts per minute. After 20 days it
gives a count rate of 500 counts per minute. What is the half-life of the material?
A 4 days B 5 days
C 20 days D 50 days

Section B [45 marks]


Answer all questions in this section.
Write your answers in the spaces provided on the question paper.

B1 Figure B1.1 below shows a simple pendulum suspended from a fixed point.

The bob is slightly pulled to position A and then released.

117 MOUNT K PUBLISHERS (0979595713)


(a) State one factor that does not affect the period of the pendulum. [1]
(b) If the values of time for 20 oscillations obtained were
16.1s, 15.9s, 16.0s, 16.2s and 15.8s, calculate the period of the pendulum. [3]
(c) Calculate the frequency of the pendulum. [2]
[Total: 6 marks]
B2 A car has a mass of 900kg. It accelerates from rest at a rate of 1.2m/s2.
Calculate the
(i) time taken to reach a velocity of 30m/s, [2]
(ii) force required to accelerate the car at a rate of 1.2m/s4. [2]
[Total: 4 marks]
B3 Figure B3.1 below is a diagram showing a load being moved using a wheelbarrow.

The total mass of the wheelbarrow and the load is 80kg. (Take g = 10N(kg)
(a) Calculate the
(i) weight of the wheelbarrow and the load, [2]
(ii) force, F, required to lift the wheelbarrow. [2]
(b) State whether force, F, would increase or reduce when the handles of the
wheelbarrow are made longer. [1]
[Total: 5 marks]

118 MOUNT K PUBLISHERS (0979595713)


B4 Figure B4.1 below is a diagram showing a stone of mass 2kg that was pushed up a slope
from Q to R.

72 joules of work was done in moving the stone up the slope from Q to R.
(a) What is the potential energy of the stone at R? [1]
(b) If the stone falls through side RT, what would its potential energy be at S, the mid-
point of its fall? [1]
(c) Calculate the
(i) height TR, [2]
(ii) velocity of the stone just before it strikes the ground. [2]
[Total: 6 marks]
B5 Figure B5.1 below is a diagram showing a wave travelling along a spring in the direction
shown.

(a) In what way is this wave the same as a sound wave?


……………………………………………………… [1]

119 MOUNT K PUBLISHERS (0979595713)


(b) What are regions U and W called?
(i) U …………………………………………….. [1]
(ii) W ……………………………………………. [1]
(c) Explain why sound waves travel faster in liquids than in gases.
……………………………………………………………. [2]
[Total: 5 marks]
B6 A ray of light travels from air into water at an angle of 40° between the normal and the
incident ray.
If this ray produces an angle of refraction of 29°, calculate
(i) the refractive index of water, [2]
(ii) the critical angle of water. [2]
[Total: 4 marks]
B7 Figure B7.1 is a diagram showing a 240V mains supply connected to a television set and
two lamps.

The power supplied to each lamp and television is 40W and 120W respectively, when the
switches are closed.
(a) Calculate the
(i) total power supplied, [2]
(ii) total number of kilowatt hours (kWh) of energy supplied to the
circuit in 3.0 hours, [2]
(iii) p.d. across the television set. [2]
[Total: 6 marks]

120 MOUNT K PUBLISHERS (0979595713)


B8 Figure B8.1 is a diagram showing a coil of wire wound on a soft iron core, with current
flowing in the direction indicated by the arrows.

(a) Mark the N and S poles induced on the iron core. [1]
(b) Show by an arrow the direction in which a plotting compass needle would point
when placed at point X. [1]
(c) A beam of electrons flows through point Y in a direction perpendicularly
downwards into the paper. Show clearly using an arrow labelled F, the direction
of the force exerted by the magnetic field on the electron beam. [1]
[Total: 3 marks]
B9 Phosphorus-32 (P-32) can decay by emitting beta particles.
(a) What is a beta particle? [1]
(b) If the proton number of phosphorus-32 is 15,
(i) state the new values of proton and mass numbers of the nuclide
just after it emits a beta particle,
Proton Number: ……………………………………. [1]
Mass Number: …………………………………….. [1]
(ii) write the decay equation for P-32 after emitting two beta particles. [2]
(c) Phosphorus-32 can be used to prove that plants absorb phosphorus nutrient from
the soil around them. State one safety precaution which should be taken into
consideration when doing experiments with phosphorus-32. [1]
[Total: 6 marks]

121 MOUNT K PUBLISHERS (0979595713)


SECTION C [20 marks]
Answer any two (2) questions from this section in the separate Answer Booklet provided.
Cl Figure C1.1 below is a velocity time graph representing motion of a motor cycle travelling
along a straight road.

(a) Describe the motion of the motor cycle between the points
(i) O and P,
(ii) P and Q,
(iii) Q and R. [3]
(b) What is the maximum speed of the motor cycle? [1]
(c) How long did it take the motor cycle to retard to rest? [1]
(d) Calculate the total distance travelled by the motor cycle. [3]
(e) Calculate the average velocity for the whole motion. [2]

122 MOUNT K PUBLISHERS (0979595713)


C2 A learner carried out a Hooke's Law experiment and obtained the following results:
Table C2.1
Mass/kg 0 0.02 0.04 0.06 0.08 0.10
Length of loaded spring / cm 11 12.1 13.2 14.3 15.4 16.5
Applied force/N
Extension/mm

(a) Copy and complete table C2.1 by finding values of applied force (N) and extension
(mm) produced. [2]
(b) Plot a graph of applied force (N) against extension (mm). [4]
(c) From the graph determine the spring constant. [2]
(d) (i) Did the spring reach its elastic limit? [1]
(ii) Explain your answer in (d)(i) above. [1]
[Total: 10 Marks]
C3 Figure C3.1 is a chart showing some components of the electromagnetic spectrum.
Blue Green microwaves
Figure C3.1
(a) (i) Use the list below to copy and complete the electromagnetic
spectrum chart.
Radio waves, x-rays, ultraviolet, gamma rays, infra-red and red light. [2]
(ii) State two properties common to all members of the electromagnetic
spectrum. [2]
(b) Name a component of the spectrum that
(i) has the longest wavelength,
(ii) is emitted by hot bodies. [2]
(c) Microwaves have a frequency of 1010 Hz and velocity of 3 x 108m/s.
(i) Calculate the wavelength of microwaves. [2]
(ii) State two practical uses of microwaves. [2]
[Total: 10 Marks]

123 MOUNT K PUBLISHERS (0979595713)


Centre Candidate
Number Number

Candidate Name

EXAMINATION COUNCIL OF ZAMBIA

Examination for General Certificate of Education Ordinary Level

Science 5124/1
Paper 1

2021

Additional information:
Electronic calculator (non-programmable) and / or Mathematical tables
Graph papers
Soft clean eraser
Soft pencil (type B or HB is recommended)

Time 2 hours

SECTION A [20 marks]


Answer all the questions on the answer grid provided in this question paper.
A1 The following dimensions of a metal sheet were measured by a welder using different
instruments

Which instruments were used to accurately measure the dimensions?


Length Thickness Width
A Metre rule Micrometre Metre rule
B Metre rule Vernier caliper Vernier caliper
C Micrometre Metre rule Micrometre
D Metre rule Engineers calipers Micrometre

124 MOUNT K PUBLISHERS (0979595713)


A2 The following is the diagram showing a simple pendulum.

What physical quantities can affect the period of the pendulum?


A The mass of the bob and the length of the pendulum.
B The length of the pendulum and acceleration due to gravity.
C Mass of the bob and acceleration due to gravity
D Acceleration due to gravity and the number of oscillations.
A3 A student weighed a small amount of sugar crystals and found the mass to be 0.568g.
After adding two small sugar crystals, the mass was found to be 0.571g. Assuming that
the sugar crystals were identical, what was the average mass of one crystal?
A 1.5 x 10-3g
B 1.5 x 103g
C 3.0 x 10-3g
D 3.0 x 103g
A4 The following graph shows how distance varies with time for a cyclist's ride that lasted for
250 seconds.

125 MOUNT K PUBLISHERS (0979595713)


What is the cyclist's average speed for the whole journey?
A 1.0m/s
B 1.2m/s
C 1.5m/s
D 2.0m/s
A5 The diagram shows a car going around a bend at a constant speed.

Which statement about the car is correct?


A There is no resultant force acting on the car
B The car is not accelerating.
C A force of constant size acts on the car towards the centre of the curve, O.
D A force of constant size acts on the car in the direction of motion.

A6 In a small hydroelectric power station, 20kg of water flows through a vertical height of
15m in one second. If the efficiency of the power station is 60% and taking 'g ' as 10N/kg,
what is the power output of the hydroelectric power station?
A 180W
B 1800W
C 3 000W
D 18 000W

126 MOUNT K PUBLISHERS (0979595713)


A7 The following diagram shows 4 identical metal cans E, F, G, H whose outside surfaces
were painted as indicated. Equal volumes of water at a temperature of 80°C were placed
in the cans.

After 5 minutes in a cool room, which metal can would contain the coolest water?
A E
B F
C G
D H

A8 A crest of a water wave travels 0.4m in 5 seconds. If the distance between two successive
crests is 0.005m, what is the frequency of the wave?
A 0.2Hz
B 1.6Hz
C 8.0Hz
D 16.0Hz
A9 An echo-sounder in an aeroplane receives an echo from the ground 20 seconds after it
was sent. If the speed of sound in air is 330m/s, how high is the plane above the ground?
A 16.5m
B 33m
C 660m
D 3 300m

127 MOUNT K PUBLISHERS (0979595713)


A10 The following diagram shows the formation of image I from object 0 using a convex lens.
(Not drawn to scale)

What is the focal length of the lens?


A 15cm
B 23cm
C 45cm
D 53cm

A11 Where is the image in a long-sighted eye formed and how can the defect be corrected?
A Behind the retina and focussed by means of a concave lens.
B In front of the retina and focussed by means of a convex lens.
C Behind the retina and focussed by means of a convex lens.
D In front of the retina and focussed by means of a concave lens.

A12 The following diagram shows the pattern and direction of the magnetic field between
two magnetic poles J and K.

Which type of poles are J and K?

128 MOUNT K PUBLISHERS (0979595713)


J K
A N-pole N-pole
B N-pole S-pole
C S-pole N-pole
D S-pole S-pole

A13 Two spheres are mounted separately on insulating stands. If both spheres are positively
charged, which diagram shows a correct electric field pattern?

A14 The diagram shows a resistor connected to a cell of e.m.f 2V.

How much heat energy is produced in the resistor in 6 seconds?


A 0.4J
B 2.5J
C 4.8J
D 10J

129 MOUNT K PUBLISHERS (0979595713)


A15 A 2kW stove, a 0.2kW television set and a 0.1kW lamp are all switched on at the same
time. What is the total cost of running these appliances for 4 hours at K2.00 per unit?
A K1.74
B K4.60
C K9.20
D K18.40

A16 A small coil is connected to a galvanometer as shown.

What happens to the pointer of the meter when the magnet is pushed into the coil? The
pointer ...
A deflects to the left and then returns to zero.
B deflects to the right and then returns to zero.
C gives a continuous reading to the left.
D gives a continuous reading to the right.
A17 Which graph shows the voltage output V against time t for an a.c generator?

130 MOUNT K PUBLISHERS (0979595713)


A18 The following diagram shows the appearance on the screen of the alternating voltage
applied to the input of an oscilloscope. The sensitivity is set to 3 volts per cm.

What is the maximum voltage?


A 1V
B 3V
C 4V
D 12V

A19 Radioactive iodine is used to treat tumours of the thyroid gland. It decays by emitting
beta particles and gamma radiation. The beta decay process is represented by the
following
equation:
131 →
53 + 0
−1 +
What is the nucleon number, A, and proton number, Z, for this nucleus of Xe?
Nucleon Proton
number number
A 78 53
B 130 55
C 131 54
D 184 78

131 MOUNT K PUBLISHERS (0979595713)


A20 A radioactive isotope has a half-life of 6 000 years. How much time passes before the rate
of emission from a sample of this isotope falls to 1/16 of the original value?
A 6 000 years
B 18 000 years
C 24 000 years
D 96 000 years

Section B [45 marks]


Answer all questions in this section. Write your answers in the spaces provided in this question
paper.
B1 Figure B1.1 is a diagram showing the reading of the Vernier callipers.

(a) What is the reading shown on the Vernier callipers?

Vernier callipers reading: ................................. [2]

(b) (i) Convert the reading in (a) above to metres.

Vernier callipers reading in metres: ....................... [2]

(ii) Write the reading in (b)(i) in standard notation.


……………………………………………………………………………………………………………… [1]
(iii) How many significant figures does the Vernier callipers reading in metres
have?
………………………………….……………………………………………………………………………[1]
[Total: 6 marks]
132 MOUNT K PUBLISHERS (0979595713)
B2 A man of mass 75kg on earth goes to the moon where the gravitational field strength is
1.6N/kg.
(a) What is the mass of the man on the moon?

Mass ......................... [1]


(b) What is the weight of the man on the moon?

Weight: ............................. [2]

(c) A light truck of mass 2 000kg accelerates from rest at a rate of 1.5m/s2.
Calculate the force required to accelerate the car at this rate.

Force ............................ [2]


[Total: 5 marks]
B3 Figure B3.1 is a diagram showing a uniform meter rule freely pivoted at its centre. A
newton metre is hung 20cm from the pivot while a 4.0N weight is suspended 40cm from
the pivot.

The metre rule is in equilibrium.


(a) State the principle of moments for a body in equilibrium.
………………………………………………………………………………………… [1]

133 MOUNT K PUBLISHERS (0979595713)


(b) Calculate the reading on the newton meter.

Reading ....................................... [2]


(c) State with a reason whether the newton meter reading would increase or
decrease when the 4.0N weight is moved to a distance of 30cm away from the
pivot.
…………………………………………………………………………………………..…………………………………
…………………………………………………………………..…………………………………………………….. [2]
[Total: 5 marks]

B4 A small hard ball of mass 0.14kg is thrown vertically upwards and reaches a height of 12m
above the point from which it is thrown.
(a) Calculate the work done in lifting the stone to the height it attains.

Work done ............................... [2]

(b) What is the velocity of the ball at the maximum height?

Velocity:.................................. [1]
(c) On a windy day the same stone is thrown upwards with the same energy, but fails
to reach the height reached earlier. Why does it happen like this?
……………………………….……………………………………………………………………………………….. [2]
[Total: 5 marks]
B5 Figure B5.1 shows transverse waves. Displacement (m).

Figure B5.1
134 MOUNT K PUBLISHERS (0979595713)
(a) Find the number of waves in Figure B5.1
Waves: ........................... [1]
(b) What is the amplitude of the waves generated in the figure above?
Amplitude............................ [1]

(c) Calculate the speed of the waves above if the number of waves in Figure B5.1 are
produced in 2 seconds.
Speed: ............................. [2]
[Total: 4 marks]
B6 Figure B6.1 shows light in air striking the vertical side of a rectangular glass block at an
angle of incidence of 60°.

Figure B6.1
The refractive index of glass is 1.6.
(a) What is refractive index?
………………………………….……………………………………………………………………………………… [1]
(b) (i) Calculate the angle of refraction, r.
Angle r ................................ [2]
(ii) What is the critical angle of the glass block?
Critical angle ............................ [2]
[Total: 5 marks]
B7 (a) Explain what is meant by
(i) magnetic field,
……………………………………………………………………………………………………………… [1]
(ii) electric field
…………………………………..………………………………………………………….……………… [1]

135 MOUNT K PUBLISHERS (0979595713)


(b) Complete the diagram in Figure 87.1 (b) to show the pattern and direction of the
magnetic field lines in the space around the bar magnet.

Figure B7.1 (b)

(c) Figure B7.1 (c) shows a positively charged sphere S placed near to an initially
uncharged isolated conductor AB. Complete the diagram to show the charges
induced in the diagram.

Figure B7.1 (c)


B8 Figure B8.1 shows an electrical circuit containing two resistors.

Figure B8.1
(a) On Figure B8.1, draw on the circuit to show how a voltmeter will be used to
measure the potential difference across the 6Ὠ resistor. [1]
(b) When the switch S is closed, calculate the current through the ammeter, A.

Current: .................... [2]

136 MOUNT K PUBLISHERS (0979595713)


(c) What is the potential difference across the 6Q resistor?
P.d: ........................... [2]
[Total: 5 marks]
B9 Uranium 238 decays to form a nucleus of thorium by emission of an alpha particle.
92

90 ℎ
Thorium has the symbol 234
(a) What is the meaning of nucleon number?
………………………………..………………………………………………………………………………………[1]
(b) Write a decay equation to show how uranium 238 decays to thorium 234 ℎ
92 90

after emitting an alpha particle. [2]

(c) 90 ℎ decays to an isotope of protactinium, Pa, by beta decay. Write


Thorium, 234
the decay equation to show this process. [2]
[Total: 5 marks]

SECTION C [20 marks]


Answer any two questions from this section in the separate Answer Booklet provided.
C1 Figure C1.1 shows a conveyor belt driven by an electric motor carrying suitcases into a
trailer of a truck.

Figure C1.1

137 MOUNT K PUBLISHERS (0979595713)


(a) Give two differences between mass and weight. [2]
(b) If a suitcase of mass 20kg is lifted from the ground to the truck. Taking 'g' as
10N/kg , calculate the
(i) weight of the suitcase, [2]
(ii) gravitational potential energy of the suitcase at the top of the conveyor as
it enters the truck. [2]
(c) The suitcase takes 12 seconds to travel 9.0m along the conveyor belt. Calculate
the power of the motor. [2]
(d) The electric motor is powered by an input voltage of 230V and current of 1.5A.
Calculate the electrical energy input to the motor in 12 seconds. [2]
[Total: 10]
C2 The Figure C2.1 shows a square block of glass JKLM with a ray of light incident on side 3K
at an angle of incidence of 60°. The refractive index of the glass is 1.50.

Figure C2.1
(a) Calculate the angle of refraction of the ray. [2]
(b) Calculate the critical angle for a ray of light in this glass. [2]
(c) Explain why the ray shown cannot emerge from KL but will emerge from side LM. [2]
(d) Diamond has a refractive index of 2.42. The speed of light in a vacuum (or in air) is
300 000km/s.
Calculate the
(i) speed of light in diamond, [2]
(ii) critical angle for diamond. [2]
[Total: 10 Marks]

138 MOUNT K PUBLISHERS (0979595713)


C3 The atomic number of americium-241 is 95. Americium-241 decays by losing an alpha
particle.
(a) (i) Explain why smoke detectors containing americium -241 are not harmful
to people. [1]
(ii) Copy and complete the equation showing this decay by giving values for X
and Y. [2]
241 →
95
4
2 + + energy

(b) Figure C3.1 shows how the activity of a sample of americium-241 changes over a
long period of time.
Time/years Activity/counts per minute
0 64
500 30
1 000 14
1 500 6
2 000 2
Figure C3.1
(i) Use these numbers to draw a graph of activity against time for
americium-241. [4]
(ii) Use your graph to find the half-life of americium-241. [1]
(c) Californium-241 is also radioactive and decays by losing an alpha particle.
It has a half-life of 4 minutes. Suggest why it would be unsuitable for use in a
smoke detector. [2]
[Total: 10 Marks]

139 MOUNT K PUBLISHERS (0979595713)


Centre Candidate
Number Number

Candidate Name

EXAMINATION COUNCIL OF ZAMBIA


Examination for School Certificate Ordinary Level

Science 5124/1
Paper 1

Tuesday 16 NOVEMBER 2021


Time: 2 hours Marks: 80

SECTION A [20 MARKS]


Answer all the questions on the answer grid provided.

A1 When converted to a base unit in scientific notation, 2GB is.


A 2 x 10-9 B.
B 2 x 10-6 B
C 2 x 10 6 B
D 2 x 10 9 B
A2 Which of the following is a derived unit?
A Ampere
B Kelvin
C Meter
D Pascal
A3 A Car moving at 3m/s increased its velocity uniformly to 9m/s in 4s. Find
the uniform acceleration of the car.
A 1.5m/s
B 2.25m/s
C 3.0m/s
D 6.75m/s

140 MOUNT K PUBLISHERS (0979595713)


A4 A rock weighed 8.0N on the moon where the acceleration due to gravity is
1.6N/kg.
Calculate the mass of the rock on earth where the force of gravity is
10N/kg.
A 0.8kg
B 5.0Kg
C 12.8kg
D 80.0kg

A5 The following diagram shows a pilot in air after jumping from a plane.

As he falls, how will the size of the forces acting on him change before he
attains terminal velocity?
Downward force Upward force Resultant force
A Increases Decreases Increases
B Increases Increases Increases
C Decreases Increases Decreases
D Decreases Decreases Decreases

A6 The following diagram shows a 30cm spanner used to turn a nut with a
force of 60N.

Find the moment of the force.


141 MOUNT K PUBLISHERS (0979595713)
A 1.8Nm
B 2.0Nm
C 200Nm
D 1 800Nm

A7 The following diagram shows a thermos flask

Identify the part(s) that prevent heat loss by convection.


A E only
B F only
C E and G
D Fand G

A8 Which of the following is not true about dull black surfaces? They are good

A absorber of heat.
B emitters of heat.
C reflectors of heat.
D radiators of heat.

142 MOUNT K PUBLISHERS (0979595713)


A9 The following diagram shows an object placed in the path of a beam of
light.

What terms best describe the shadows H, J and K?


H J K
A Umbra Penumbra Umbra
B Penumbra Umbra Umbra
C Umbra Umbra Penumbra
D Penumbra Umbra Penumbra

A10 Which of the following is true about images formed by plane mirrors?
They are….
A diminished.
B magnified.
C upside down.
D Virtual.

143 MOUNT K PUBLISHERS (0979595713)


A11 The following diagram shows an image produced by a biconvex lens. (The
diagram is drawn to scale).
Which position A, B, C or D shows the correct position of the object?

A12 The following diagram shows a ray of light emerging from water into air.

which labelled arrow shows the direction of the ray in air?

A13 A siren from an ambulance of police car has a very high pitch. Which of the
following can change the pitch of a sound wave?
A Amplitude
B Frequency
C Velocity
D Wavelength

144 MOUNT K PUBLISHERS (0979595713)


A14 Which of the following materials is most suitable for making permanent
magnets?
A Brass
B Copper
C Iron
D Steel

A15 The leaves of a positively charged electroscope diverged when a charged


body was
brought near it. This proves that the body was
A positively charged.
B negatively charged.
C neutral of charge.
D a good conductor.

A16 The following circuit diagrams show three bulbs of equal resistance
connected to a 6V power supply. Which diagram will all the bulbs light
brightest?

A17 A hair dryer is made of two heaters with a power rating of 400W each and a
fan motor with a power rating of 100W.
Calculate the cost of using the hair dryer for two hours if a unit of electricity
costs 50n.
A K0.45
B K0.90
C K9.00
D K90.00
145 MOUNT K PUBLISHERS (0979595713)
A18 The following diagram shows a piece of steel placed inside a solenoid.

If the switch, S, is closed and opened several times, the end A of the steel
bar
Becomes…
A temporal north pole.
B Permanent north pole.
C permanent south pole.
D temporal south pole.

A19 What is the use of the hot filament in a Cathode Ray Oscilloscope (CRO)? It
is to…
A heat the anode.
B heat the cathode.
C produce light.
D Produce protons

A20 Strontium 90 (90


38Sr) undergoes a beta decay to produce element Y.
Which of the following shows the decay equation?
A 90Sr → 90 0
38 37Y − 1e
B 90Sr → 90
38 38
Y + 01e
C 90Sr → 90
38 39
Y + 01e
D 90Sr → 90
38 39Y − 01e

146 MOUNT K PUBLISHERS (0979595713)


Section B [45 marks]
Answer all questions in this section.
Write your answers in the spaces provided in the question paper.

B1 The following graph shows a distance/time graph of a bus moving from


town A to town B.

(a) What is the total distance between town A and town B?


…………………………………………………………………………………………… [1]
(b) The bus stopped at a station between town A and town B. For how
long did the bus stop at this station?
……………………………………………………………………………………………… [1]
(c) Calculate the speed of the bus between town A and the first station.

Speed = ..................................... [2]

(d) Calculate the average speed for the whole journey.

Average speed =........................................ [2]

(e) Mention one consequence of over-speeding.


………………………………………………………………………………………………… [1]
[Total: 7 marks]

147 MOUNT K PUBLISHERS (0979595713)


B2 (a) State Newton's Second Law of motion.

…………………………………………………………………………………………………………… [1]
(b) The following diagram shows a 4kg box resting on a frictionless
surface with forces acting on it as shown.

Calculate the uniform acceleration caused by the forces.

Acceleration = ...................................... [2]

B3 The following diagram shows how energy flows into and out of a coal-fired
power
station.

(a) How much useful electrical energy is produced by the power station?

Useful energy = .............................................. [1]

148 MOUNT K PUBLISHERS (0979595713)


(b) Calculate how much energy is wasted from the cooling tower.
Wasted energy = …………………………………………. [2]
(c) Calculate the efficiency of the power station in producing electricity.
Efficiency = .................................. [2]
[Total: 5 marks]
B4 (a) Define convection.
………………………………………………………………………………………………………
[1]
(b) The following diagram shows an incomplete diagram of convection
currents occurring in nature at night.

(i) Name the breeze.


………………………………………………………………………………………. [1]
(ii) On the diagram, draw convection currents for this breeze.
[Total: 4 marks]

B5 (a) List the components of the electromagnetic spectrum in order of


Increasing frequency. (Least to highest).
………………………………………………………………………………………………. [2]
(b) Mention one Similarity among the components of the spectrum.
……………………………………………………………………………………………….. [1]
(c) Mention one difference among the components of the spectrum.
……………………………………………………………………………………………….. [1]
(d) Give one harmful effect of the spectrum,
[Total: 5 marks]

149 MOUNT K PUBLISHERS (0979595713)


B6 The following diagram shows an ultrasound source sending a sound wave
into the human body.

Figure B6.1
The total time for the ultrasound to travel in the bone from X to Y and back
to X is 9.0 x 10 -6 s.
(a) What does the term ultrasound mean?
………………………………………………………………………………………………. [1]
(b) If the speed of the ultrasound in bone is 4 100m/s, calculate Z the
thickness of the bone.

Z = ........................................................ [2]

(c) Mention another use of ultrasound other than medical use.


………………………………………………………………………………………………. [1]
[Total: 4 marks]

B7 (a) Define reflection of light.


………………………………………………………………………………………………. [1]

150 MOUNT K PUBLISHERS (0979595713)


(b) The following diagram shows a dog and a cat standing in front of a
plane mirror.

Using the diagram, show by construction, how the dog sees the cat's image
(c) Give two characteristics of the images formed by plane mirrors.
………………………………………………………………………………………………………….
……………………………………………………………………………………………….. [2]
[Total: 6 marks]
B8 The following diagram shows a graph of voltage against current of a
conductor.

151 MOUNT K PUBLISHERS (0979595713)


(a) what is the maximum current through the conductor?
Current = ........................................ [1]
(b) Calculate the resistance of the conductor when the current was 1.0A.

Resistance = ................................................... [2]


(c) Calculate the power in the conductor when the voltage was at 6v.

Power = .................................... [2]


Total: 5 marks]

B9 (a) Define nuclear fission.


………………………………………………………………………………………………… [1]
(b) Uranium-235 (235 92 ) when bombarded by slow moving neutron
disintegrates into barium and Krypton plus two neutrons.
(i) What term is used to describe the products of nuclear fission?
……………………………………………………………………………………. [1]
90
(ii) Given that the products of this reaction are 144
56 and 36
plus two
neutrons, Write the equation for this reaction.
……………………………………………………………………………………. [1]
(c) Mention one harmful effect of fission reactions.
……………………………………………………………………………………………….. [1]
(d) Give one advantage of nuclear reactors as a Source of energy.
……………………………………………………………………………………………….. [1]
[Total: 6 marks]

Section C [20 marks]


Answer any two questions from this section in the separate Answer Booklet
provided.

152 MOUNT K PUBLISHERS (0979595713)


C1 (a) Describe an experiment on how you can determine the lower fixed
point of a thermometer. [2]
(b) Give two differences between a laboratory thermometer and clinical
thermometer.
[2]
(c) Mention one way in which you can improve the
(i) sensitivity of a thermometer,
(ii) accuracy of a thermometer. [2]

(d) (i) Name a device that can be used to measure very high
temperatures and those that vary considerably. [2]
(ii) Mention two devices that use the device you have mentioned
in (d) (i) above.
[Total: 10 marks]

C2 The following data was obtained from an experiment to determine the


focal length of a biconvex lens.
U (cm) 30.0 35.0 40.0 45.0 50.0
V (cm) 44.0 37.0 32.0 31.0 28.0
V
( )
U

(a) Copy and complete the table to get all the values of M (magnification) [2]

(b) Plot a graph of M against V. [4]


(c) Determine G the slope of the graph. [2]
(d) Use the slope G of your graph to determine f (focal length) of the
lens using
= −1 [1]
(e) Mention one use of this type of lens. [1]

[Total: 10 marks]

153 MOUNT K PUBLISHERS (0979595713)


C3 (a) State Hooke's Law. [1]
(b) The following table shows the readings of a pointer of a scale for
different masses attached total different intervals.

Mass (kg) 0 0.2 0.4 0.6 0.8


Pointer reading (mm) 120 126 132 138 144
Force (N)

(i) Copy and complete the table to determine the force acing on
each mass on the spring. [2]

(ii) Plot a graph of force against extension. [4]

(iii) Determine the extension for a force of 5N. [2]

(c) Differentiate between plasticity and elasticity. [1]


[Total: 10 marks]

154 MOUNT K PUBLISHERS (0979595713)


ANSWERS FOR SCIENCE PAPER 1 - 2015 (PHYSICS PART)
1. A 6. C 11. B 16. B
2. C 7. C 12. D 17. D
3. C 8. B 13. A 18. A
4. A 9. D 14. B 19. C
5. C 10. A 15. C 20. A

ANSWERS FOR SCIENCE PAPER 1 (PHYSICS PART) – 2016


1. D 6. C 11. B 16. C
2. A 7. A 12. A 17. D
3. C 8. A 13. B 18. C
4. B 9. D 14. C 19. D
5. X. 24.49m/s 10. X. 0.083Hz 15. A 20. C

ANSWERS FOR SCIENCE PAPER 2 (PHYSICS) – 25 JULY 2016


ANSWERS FOR SECTION A [45 MARKS]
1 (a) Oscillating is the swinging of the bob
0.20
(b) T= = = 0.4s
0.5
1 1
(c) f= = = 2.5Hz
0.4

2 (a) (i) electronic balance


(ii) mass = 200kgm
(b) Weight = mg = 200 x 10 = 2000N
3 (a) Moment is a turning effect of force about a certain point
(b) centre of mass; size of base area
(c) F1d1 = F2d2 » 3.15 x 0.1 = F2 x 0.3 » F2 = 1.05N.
4 (a) Gravitational potential energy
(b) X = ut + ½gt = (0x2) + (½ x 10 x 22) » X = 20m
(c) (i) K.E = ½mgh = ½ x 2 x 10 x 20 = 200J
(ii) V = u + gt = 0 + 10 (2) = 20m/s
5 (a) Convection / Radiation
(b) Metal plate B
(c) Metal A, because silvery surface is bad emitter of heat.
6 (a) Longitudinal waves
(b)
KEY
C – Compression
R – Rarefaction

(C) the speed of sound will increase.


7 (a) A – Incident ray,
B – Refracted ray,
C – Emergent ray and
D – Reflected ray
(b) Angle between ray A and normal = i and angle between ray B and normal is r (mark
these angles)
sin sin 25 0.4226
(c) n = sin = sin 16
= 0.2756
= 1.53

8 (i) Aluminium is a non – magnetic material so it cannot magnetized hence no


attraction of iron nails.
(ii) Iron – when the switch is ON, it will easily magnetize and more iron nails will be
attracted to it and when the switch is OFF, it will easily demagnetize and lose all
the iron nails.
(iii) Steel - when the switch is ON, it will hardly magnetize and small amount of iron
nails will be attracted to it and when the switch is OFF, it will hardly demagnetize
and lose some iron nails.
9 (a) A – Beta radiation
B – Gamma radiation
C – Alpha radiation
(b) It has no charge (neutral)

228 +
(b) 232
90 ℎ −→ 88
4
2
ANSWERS FOR SECTION B [20 MARKS]
10 (a) Incline plane – it moves or lifts loads
Opener – opening lids on bottle tops
(b) (i) Because some energy will be lost in form of heat energy.
(ii) Because some energy will be lost in form of heat energy.
(c)

60
(d) (i) M.A = = =3
20
. 3
(ii) Efficiency = X 100 = X100 = 75%
. 4

11 (a) (1) Transverse waves have crest and troughs while Longitudinal waves have
compression and rarefaction.
(2) Transverse waves the vibration of particles are at right angle to the
direction of a wave while Longitudinal wave the vibration of particles are
parallel to the direction of a wave.
(b) Get a ripple tank and in the water place a vibrating bar which is connected to the
power pack. Down a ripple tank put a white paper and on top of a ripple tank put
a lamp. Then place a metal plate (barrier) in the ripple tank at an angle of 90 o to
the direction of the wave.
(c) (i) T = t ÷ n = 3 ÷ 2 = 1.5s
(ii) ʎ = f ÷ V = 0.2 ÷ 0.67 = 0.300 m
12 (a) Electric charge is the product of current with time while electric current is the rate of
flow charge.
(b) (i)

6
(ii) R = = =
1.2

2+ 1 3+2
(c) (i) R= = = 2.4Ω
2 2 2 3

12
(ii) I= = = 5A
2.4
ANSWERS FOR SCIENCE PAPER 1 – 5 NOVEMBER 2016
SECTION A [20 MARKS]
1. A 6. D 11. B 16. D
2. B 7. B 12. B 17. B
3. C 8. D 13. D 18. D
4. C 9. A 14. A / D 19. C
5. C 10. B 15. C 20. A

SECTION 45 [45 MARKS]


1 (a) Micrometer Screw gauge
(b) Sleeve reading: 8.50 mm
Thimble reading: 22 x 0.01mm = 0.22mm
Final reading: 8.50mm + 0.22mm = 8.72mm
(c) 1 Clean the anvil and spindle of the instrument.
2 Zero the instrument before it is used i.e. when the instrument is closed,
the zero mark of the thimble and that of the sleeve must coincide.

1 1
2 (a) ℎ = 2 = × 10 × 82 = 350m
2 2

(b) V2 = 2gh = 2 x 10 x 350 = 7000m/s ---> V = 83.666m/s


2 1.0 ×7000 /
Kinetic energy = = = 3 500 J
2 2
(c) Force = mass x retardation
V2 = u2 + 2ax --> 02 = 83.666 + 2 x a x 0.05 ---> - 83.666 = 0.1a
2
a = - 836.66m/s --> Retardation = 836.66m/s2
Force = mass x retardation = 1.0 x 836.66 = 836.66N

3 P1 = 99000Pa, T1 = 27oC = 300K, P2 = ???, T2 = 5oC = 278K


1× 2 99000 × 278 27 522 000
2= = = = 91 740Pa
1 300 300

9600
4 (a) . = = =4
2400
(b) V.R = 5
.
(c) = .
× 100 = 4 × 100 = 80%
5
(d) Distance by effort = Velocity ratio x distance by load = 5 x 20 = 100m
3
5 (a) wavelength (ʎ) = = 2cm or 0.02m
1.5
0.06
(b) ( )= = 1.5 = 0.04s
1 1
(c) V =fʎ = × ʎ = × 0.02 = 0.5m/s
0.04

6 (a) Range is -19oC to 110oC


(b) The more sensitive thermometer is quick to detect temperature changes while a
less sensitive thermometer takes time to detect temperature changes.
(c) It has a large bulb and a narrow bore (capillary tube).
(d) A clinical thermometer has a constriction and short range from 35oC to 42oC while
laboratory thermometer has no constriction and has large range 19 oC to 110oC.

2 × 1 12 × 4 48
7 (a) Effective resistance (R) = = = = 3Ω
1 + 2 4 +12 16
6
(b) (i) = = = 1.5A
4
6
(ii) = = = 2A
3
(iii) The current will increase because the total resistance will increase cause
the reciprocal of the total resistance is equal to the sum of the reciprocal
of individual resistances.

8 (a) =
×
=
240 ×25 = 12V
500
12 × 8 96
(b) = × 100 = × 100 = × 100 = × 100
240 × 0.6 144
Efficiency = 66.67%
9 (a) 139 − −→ 135 + 42
57 55
(b) 135 − −→ 135 + 0
55 56 −1

SECTION B [20 MARKS]


1 (a) AIM: To find the centre of gravity.
Apparatus: Irregular shaped piece cardboard, Pen, String
Procedure:
Make three holes near the edge of the irregular shaped piece of cardboard. Three
holes are as far as possible. Cut the string into two, short one(5cm) and long
one(15cm). Make the plumb by the long string and a pen. Thread a hole with the
short string. Hang the plumb on the short string. Draw the line of the plumb on
the paper after it is steady (The paper must be free to move at the hanging point.)
Find the intersection of three lines. Make sure that the paper balances by putting
a finger at the point of intersection.
Conclusion: Since the centre of mass lies on each of the lines, the intersection
locates the centre of mass.
(b) (i) Centre of mass at 50cm mark since it’s a uniform metre rule.
(ii) Weight of rule (F ): ×
= 1 1=
0.08 × 0.3 = 0.12N
2 2 0.2
2
2 (a)
1 1
(ii) =( )  25 =( )  0.08475 = 0.5n
0 2 295 2

log 0.08475 = log 0.5 x n  -1.072 = - 0.301n  n = 3.56


20
= → = = = 5.6179
3.56

Half-life = 5.62minutes
5
1 1 1 1.5 1 5
(c) =( )  =( )  1.5 = ( )1 → =( )
0 2 0 2 0 2 0 2
1
1.5 =( ) → = 1.5 × 32 =
32 0
0

The decayed mass = 48 – 1.5 = 46.5g


(d) (i) Advantage
Gamma radiation emitted has the highest speed (moves at the speed of
light) and highest penetrating power which can be used to trace the
leakage in a pipe very fast (at a shortest period of time).
Disadvantage
Gamma radiation emitted, if it goes direct contact with human beings it
causes cancer.
(ii) A radioactive with shortest period of half – life like one minute fast
detection of leakage in the pipe.

ANSWERS FOR SCIENCE PAPER 1 – 31 JULY 2017


ANSWERS FOR SECTION A [20 MARKS]
1. B 6. B 11. D 16. B
2. A 7. C 12. B 17. D
3. D 8. B 13. A 18. D
4. A 9. C 14. C 19. C
5. B 10. C 15. A 20. A
ANSWERS FOR SECTION B [45 MARKS]
1 (a) M.R = 7.90 cm
V.R = 0.05 cm
F.R = 7.95 cm
(b) S.I units in words S.I units in symbols
(i) Metre per second m/s
(ii) Kelvin K
(iii) Metre per second squared m/s2

2 (a) Relative density has no unit while density has units.


(b) (i) V = lbh = 5 x5 6 = 150cm3
(ii) V = 5 x5 x2 = 50 cm3
80
(iv) q= = = 1.6g/cm3
50

3 (a) Simple machine is any device by means of which a force applied at one point can
be used to overcome a force at a different point.
480
(b) M.A = = = 2.4
200

. 2.4
(c) Efficiency = X 100 = X 100 = 80%
. 3

4 (a) Air molecules will be heated which will cause them to expand hence pushing the
piston upward.
(b) (i) Volume reduces which causes the increase of collision of gases hence high
pressure.
(ii) P1 x V1 = P2 x V2 » 360 x 60 = P2 x 20 » P2 = 1080 Pa

5 (a) N = X – ray
(b) it is used to microwave oven.
(c) They can pass through a vacuum.
(d) Cobalt (Co-60) covered with aluminium
6 (i)

Size of the image = 1.6 cm tall


Position of the image = 3.40 cm from the optical centre

ℎ ℎ 1.6
(ii) = = = 0.8x
ℎ ℎ 2.0

7 (a) R – Neutral point


(b) Q – north pole, T – South pole
(c) They prevent the magnetic field lines from coming out of the magnet.
(d) 1 Iron is easy to magnetize while steel is hard to magnetize.
2 Iron is easy to demagnetize while steel is hard to demagnetize.

8 (a) Alpha particle is a radiation whose nature is like that of helium atom with a mass of 4
a.m.u and a charge of +2.
206 +
(b) 210
84 −→ 82
4
2

(c) Alpha – is used to detect smoke in industries.

2 1 12 6
9 (a) R= = = 4Ω
1+ 2 6+12
12
(b) I = = = 2A
6
6
(c) I = = = 1A
6
ANSWERS FOR SECTION C [20 MARKS]
1 (a)

(b) V = ut + at » 9 = 0 + a (30) » a = 0.3m/s2


(c) F = ma 2000 x 0.3 = 600N

(d) X = l x b = 500 x 9 = 4500m


4635
(e) Average speed = = = 8.75m/s
530

2 (a)
(b)

(c) Non – Ohmic because the graph has not obeyed ohm’s law. (the graph is not straight)
(d) (i) Potential difference = 26.5 V
(ii) R = = 26.5 = 10.3846 = 10.19 Ω
2.6
3 (a) A beta particle is a particle whose nature is like that of an electron with zero
mass and a charge of -1.
(b) Nucleon number = 223 and Proton number = 98
(c) (i)
1 1 1
(c) (ii) = (1) → 40 =( ) → =( ) → 0.1 = 0.5n
0 2 400 2 10 2
−1.0
log 0.1 = log 0.5 x n ---> -1.0 = - 0.301n ---> = −0.301 = 3.32

= → = × = 3.32 x 4 = 13.2890 = 13.29 Years

(iii) Cobalt (Co-60) covered with aluminium

ANSWERS FOR SCIENCE PAPER 1 – 8 NOVEMBER 2017


SECTION A [20 Marks]
1. B 6. D 11. C 16. B
2. A 7. D 12. B 17. C
3. B 8. A 13. B 18. C
4. C 9. B 14. A 19. C
5. B 10. A 15. B 20. A

SECTION B [45 Marks]


0.5
B1 (a) = = = 1 second
0.5

0.5
(b) = = = 1 Hz
0.5

(c) (i) Frequency will decrease because frequency is inversely


proportional to length of the pendulum
(ii) Frequency will remain the same because mass doesn’t affect the
period or frequency of the pendulum.

B2 (a) A car decelerated non – uniformly from 3.33m/s to 3m/s in 5seconds and covered a
distance of 10 metres.
(b) Points A and B or Points B and C or Points A and C
(c) (i) Distance = 10m
60
(ii) = = = 3m/s
20
B3 (a) Extension = Length of loaded spring minus original length of the spring
4cm = 20 – x -------> Original length = 20 – 4 = 16cm

8 10 4 × 10
(b) = → 4
= → = = 5cm
8

B4 (a) P.E = mgh = 50 x 10 x 100 = 50 000 J


(b) K.E = 50 000J
Reason: All potential energy was converted to kinetic energy because energy cannot
be created or destroyed but it can be converted from one form to another.
1 1 1
(c) = + 2 → = + 2 → 100 = (0 × ) + × 10 2
2 2 2

100 = 0 + 5 2 → 2 = 20 → = .

B5 (a) (i) Thermometer A or Thermometer D


(ii) State: Thermometer B
Explanation: Because it can measure smallest temperature (-10oC).
(b) (i) Thermocouple is very sensitive to temperature changes.
(ii) Thermocouple can measure highest temperatures (1500oC).

B6 (a) Transverse wave


8
(b) = = = 4 second
2
1 1
(c) ( )= = = 0.25 Hz
4

(d) 20 cm
(e) (i) They both transfer energy in form of waves
(ii) They both travel faster at high temperature.
B7 (a)

(b) =

3= → = 3 ×2 → = 6.0cm
2

Therefore, distance of image from lens = 6.0 cm


1 1 1 × 2 × 6 12
(c) = + → = → = → = = 1.5cm
+ 6+ 2 8

B8 (a) Power = Voltage x current = 230 x 3.5 = 805W


230
(b) = = 3.5
= 65.714Ω
× × 2 65.714 × 2 (65.714)
Total resistance = = =
+ 2 + 2(65.714) + 65.714

65.714 ×131.428 8636.66


= = = 43.81Ω
131.428 + 65.714 197.142
B9 (i) Alpha particle is a radiation whose nature is like that of helium atom with a mass of
4 a.m.u and a charge of positive 2 (+2).
117 113
(ii) 58 56 + 4
2

SECTION C [ 20 Marks]
C1 (a) V = u + at = 0 + 0.5 x 20 = 0 + 10 = 10m/s
(b)

10
Velocity (m/s)

0 20 40 60
Time (s)
− 0−10 −10
(c) (i) = = = = - 1m/s2
10 10

Therefore, retardation = 1m/s2


1 1
(ii) = ( + )ℎ = (30 + 60)10 = 5 × 90 = 450m
2 2

C2 (a) Electrical energy


(b) Instrument P: Voltmeter
Instrument Q: Ammeter
(c) Function of S: It is used in a circuit to vary the control of electric current.
(d) (i)

2− 1 4.1−1.1 3.0
(d) (ii) Resistance (Gradient) = = = = 5.46Ω
2− 1 0.75−0.20 0.55

C3 (a) Isotopes are atoms of the same element having the same number of
protons but different numbers of neutrons. OR Isotopes are atoms of the same
element having same atomic number but different mass numbers
(b) (i) Number of electrons = 38
(ii) Number of Neurons = 90 – 38 = 52
90 90 0
(c) 38 39 + −1

87
1 1 1
(d) =( ) → = ( )29 → = ( )3
2 6.0 × 108 2 6.0 × 108 2

13 → 1 8
= (23) = 8 -----> 8A = 6.0 x 10
6.0 × 108 6.0 × 108

A = 7.5 x 107 = 75 000 000 atoms of stronthium

(e) It is random because it is a matter of pure chance whether or not a particular atom
will decay during a certain period of time.

ANSWERS FOR SCIENCE PAPER 1 – 18 NOVEMBER 2018


SECTION A [20 MARKS]
1. D 6. C 11. C 16. A
2. C 7. C 12. B 17. D
3. A 8. A 13. D 18. B
4. D 9. B 14. C 19. B
5. C 10. A 15. A 20. A

SECTION B [45 MARKS]


1. (a) Volume of liquid M
V = lbh = 0.6 x 0.5 x 0.4 = 0.12m3
Volume of liquid N
V = lbh = 0.6 x 0.5 x 0.2 = 0.06m3
(b) Mass = (1 ÷ 0.12) x 80 = 666.67kg
50
(c) = =
0.06
= 833.33kg/m3

2. (a) Total drag force = 2 000 N


(b) (i) FN = F – f = 3200N – 2000N = 1200N
800
(ii) = =
1200
= 0.67m/s2
3. (a) W = mg = 1.2 x 10 = 12N
(b) K.E = P.E = mgh = 1.2 x 10 x 120 = 1 440 J
4. (a) Load = weight = mg = 8 x 10 = 80N
80
. = = = 2.
40
. 2
(b) = × 100 = × 100 = 66.67%
. 3

(c) Because some energy is lost through friction in form of heat energy
5. (a) d = 17m, t = 50 x 10-3 seconds
17
v = = = 340 m/s
50 ×10−3

(b) Distance = speed x time = 340 x (80 x10-3) = 27.2 m


6. (a)

(b) Angle of incidence (i) = 34o and angel of refraction (r) = 25o
sin sin 34 0.559
= = = = 1.32
sin sin 25 0.423

(c) Light ray must travels from an optically denser medium to an optically less dense
medium and the angle of incidence must be greater than the critical angle.
7. (a) (i) Liquid state
(ii) Liquid state
(b) (i) By covering the tank with a material that can prevent the
loss of heat by convection.
(ii) non –metal materials e.g. plastic, wood, polyethene, etc.
8. (a) I = 0.5A, t = 2minutes = 120 seconds
Q = It = 0.5 x 120 = 60C
(b) Energy = voltage x charge = 12 x 60 = 720 J
12
(c) = = = 24Ω
0.5

9. (a) Radioactivity is the spontaneous or random disintegration of a particle by the


emission of radiation.
232
(b) 90 ℎ → + 2 −10
(c) Protons: Neutrons:
90 = a + (-2) N=A–Z
90 = a – 2 N = 232 – 92
90 +2 = a N = 140
92 = a
a = 92. Therefore, protons = 92.

SECTION C [20 MARKS]


1. (a)
(b) (i) a = 5, b = 20, h = 30
( + )ℎ (5+20) 30 750
= = = = 350m
2 2 2

(ii) a = 15, b = 40, h = 30


( + )ℎ (15+40) 30 1650
= = = = 825m
2 2 2
825
(iii) = = = 20.625m/s
40

2. (a) Thermal expansion is the increase in size of a substance due to heat.


(b) (i) T = 273 + oC = 273 + 50 = 323K
(ii) Its sensitivity
(iii) 1 Mercury has a high boiling point (357 oC) while alcohol It has a
low boiling point (78 oC).
2 Mercury is silvery coloured and doesn’t allow light through (It
is easy to see) while alcohol is colourless but is made visible by adding
coloring.
3 mercury does not wet glass. (It doesn’t stick to glass) while alcohol stick
to the wall of the capillary tube when the thread is falling.
10 1000
(c) = × 100 = = 33.33 oC
30 30

(d) Thermocouple thermometer


(e) Thermostat
3. (i)
Time in years 0 1 2 3 4 5 6
Recorded activity in count /min 124 80 52 34 23 16 12
Activity due to sample 120 76 48 30 19 12 8

(ii) Background radiation is the radiation emitted by the surroundings.


(ii) Papers, bricks, concrete, iron sheets etc.
(iii)

ANSWERS FOR SCIENCE PAPER 1 – 2 August 2019


SECTION A [20 Marks]
1. C 6. B 11. D 16. D
2. C 7. A 12. D 17. D
3. A 8. C 13. A 18. A
4. C 9. B 14. C 19. B
5. B 10. B 15. A 20. C

SECTION B [45 Marks]


40
B1 (a) (i) V= = = 4m/s
10

(ii) v = u + at » 4 = 0 + a (10) » a = 0.4m/s2

(b) Time = 12 – 10 = 2s
B2 (a) (i) M = Fd = (3 x 10) x 0.2 = 6Nm
(ii) F1d1 = F2d2 + F3d3 » 30 x 0.2 = (F2 x 0.2) + (10 x0.5)
F2 = 5N
(b) Opening and closing a door
B3 (a) Elastic potential energy
(b) P.E = mgh = 0.2 x 10 x 1.5 = 3J
(c) Some energy are lost inform of heat and sound energy

B4 (a) Velocity Ratio (V.R) = 5


(b) First calculate for mechanical advantage (M.A)
. .
Efficiency = .
× 100 −→ 80 = 5
× 100
100 . 100 . 80 ×5
80 = −→ = −→ . =
5 100 100

Then calculate effort by using M.A which is equal to 4


200 ×10 4 2000
. = − −→ 4 = −→ =
4 4

= 500N
B5 (a) (i) Amplitude = 15m
150
(ii) Wavelength = = = 60 cm (b)
2.5

Frequency (f) = wavelength (ʎ) ÷ speed (V) = 3.0 ÷ 0.6 = 5 Hz


B6 (a) (i) Positive charge
(ii) Coulomb (C)
(b) A paper is induced by a charged Perspex disc and the paper rather polarized
therefore, the negative and positive charges move on a separate side and that side
where negative charges are, attracted to the rod because unlike terms attract.
(c) Name of charge: Negative charge
Explanation: because some electrons escape from fur and move to the polythene
hence it increases in the number of electrons therefore, it will be negatively charged
and the fur decreases in the number of electrons hence it will be positively charged.
B7 (a) Combined Resistance = R1 + R2 = 2 +3 = 5Ω
2x 3 3x6
(b) Total resistance: R = + 1 = + 2 = 4Ω
3+ 2 6+3
12
(c) Potential difference: V = IR = 2.4 x 2 = 4.8 V (I = = 2.4 )
5
12
(d) Total current: I = = =3A
4
B8 (a) 1 – increase the number of turns on the coil.
2 – Increase the power supply to the coil.
(b) Because iron is strong and easily magnetize and it easily demagnetize.
B9 (a) Nuclear fission is the splitting of a nucleus of an atom due to neutron
bombardment to release a large amount of energy.
(b) (i) Original mass = 80 Kg
(ii) Quantity remained = 10 kg
(iii) Half – life = 2 days

SECTION C [ 20 Marks]
C1 (a) (i)
(a) (ii) Thinking distance = 6.0 m
(b) (i) Thinking distance = Speed x Time = (32km/h) x (1÷3600) hrs
Thinking distance = 32 x 0.00028 = 0.00896km = 8.96m
(ii) It has increased his thinking distance

C2 (a) (i) TITLE: Real depth against Apparent depth

(a) (ii) The gradient represents: Refractive index of Paraffin


2− 1 14−4 10
(iii) Gradient = = = = 1.43
2− 1 9.8−2.8 7
sin 1.33 sin 0.354906
(b) = → = → sin = = 0.5896
sin 1.45 sin 40 1.45

Angle of incidence = sin-1 (0.5896) = 36.13o


C3 (a) (i) Particle W: Beta particle
(ii) Neurons = 24
(iii) Sodium – 24 has 13 neutrons in its nucleus whereas
Sodium – 23 has 12 neutrons in its nucleus.
(b) (i) To know the amount of background radiation.
(ii) It has not become radioactive because the number of count rate is
high before the treatment than after the treatment. This means there
were more radiations in food sample before than after.
(iii) It is used in the treatment of cancer.
(iv) - Point the source of radiation away from the people.
- Wear gloves and use tongs when handling substances that emit
radiations.

ANSWERS FOR SCIENCE PAPER 1 – 18 NOVEMBER 2019


SECTION A [20 MARKS]
1. B 6. C 11. D 16. B
2. A 7. B 12. C 17. C
3. D 8. C 13. D 18. C
4. B 9. C 14. C 19. B
5. A 10. B 15. D 20. A

SECTION B [45 MARKS]


− 2−12
B1 (a) = = = -2m/s2
5

1 1
(b) = ( + )ℎ + ( ) = (2 + 12)5 + (5 x2) = (7 x 5) + 10 = 45m
2 2

45
(c) = = = 4.5m/s
10

B2 (a) Clockwise moment must be equal to anticlockwise moment.

(b) M = Fd = 50N x 0.56m = 32.5Nm

2× 2 50 ×0.55
(c) 1 = = = 275N
1 0.1
B3 (a) USEFUL ENERGE = 300J

(b) Wasted energy = 1000 – ( 70 + 30 + 300) = 1000 – 400 = 600J

300
(C) = x 100 = x 100 = 30%
1000

B4 (a) Boyle’s law states that for a fixed mass of any gas, volume is direct
proportional to pressure provided temperature is kept constant.

1× 1 100 ×5
(b) 2= = = 200kpa
2 2.5

(c) The gas molecules are in random and continuous motion which increased when
the volume reduced.

B5 (a) (i) Upper audible frequency = 20 000Hz ÷ 1000 = 20 kHz

330
(ii) Wavelength = = = 16.5m
20

(b) (i) Letter W

(ii) Letter X

B6 (a) Snell’s law states that the ratio of the sine of the angle of incidence
to the sine of the angle of refraction is a constant.

(b) They bend away from normal.

ℎ 15
(c) n= = = 1.32979 = 1.33
ℎ 11.28

B7 (a) Series connection

(b) Total resistance = R1 + R2 = 2Ω + 2Ω = 4Ω

3
(c) = = = 0.75A
4
B8 (a) (i) a.c : Alternative current

(ii) d.c: Direct current


6 6 ×2000
(b) = −→ = −→ = = 50 turns
240 2000 240

(c)

B9 (a) Nucleon number is the sum total number of protons and that of neutrons.

(b) (i) Protons number of alpha – particle = 2


(ii) Nucleon number of alpha – particle = 4
(iii) Protons number of thorium = 92 – 2 = 90
(iv) Nucleon number of thorium = 238 – 4 = 234

SECTION C [20 Marks]

C1 (a) Hooke’s Law states that; provided the elastic limit is not exceeded, the extension
of a spring is directly proportional to the load applied on the spring.
(b)
Mass (kg) 0 0.02 0.04 0.06 0.08 1.00
Reading (cm) 10.0 12.0 14.0 16.0 17.0 17.5
Applied force (N) 0 0.2 0.4 0.6 0.8 1.0
Extension (cm) 0 2.0 4.0 6.0 7.0 7.5
(c)

240
C2 (a) (i) = = = 0.48A
200+300

(ii) . = × = 0.48 × 200 = 96V

(b)

Ω Ω
(c) (i) Current through lamp S
240
= = = 1.2A
200

Current through lamp T


240
= = = 0.8A
300

1 1 1 1× 2 200 ×300 60000


(ii) = + −→ = = = = 120Ω
2 2 2+ 1 300+200 500

(d) 1 – Voltage is supplied equally to all the lamps

2 – Even when one lamp is damaged or removed, the rest will remained ON.

C3 (a) Law of conservation of energy states that energy cannot be created


or destroyed but it can be changed from one form to another.

(b) TITLE: Potential energy against time


9 9
(c) Height = = = = 1m
10 ×(900 ÷1000) 9
×

(d) (i) P.E + K.E = 9 J  4.5 J + K.E = 9J


K.E = 9 J – 4.5J = 4.5 J
(ii) P.E + K.E = 9J  1.8 J + K.E = 9 J
K.E = 9 J – 1.8J = 7.2 J

ANSWERS FOR SCIENCE PAPER 1 – 2020 G.C.E


SECTION A [20 Marks]
1. C 6. B 11. D 16. B
2. D 7. C 12. A 17. D
3. C 8. B 13. C 18. B
4. B 9. B 14. D 19. A
5. C 10. No answer 15. C 20. B

SECTION B [45 Marks]


B1 (a) Mass = 75kg
(b) Weight = mass x gravity = 75kg x 1.6N/kg = 120N
(c) - S.I unit for mass is kilogram (kg) while S.I unit for weight is newton (N)
- Mass is the amount of matter contained in a object while weight is the pull of gravity
- Mass does not change (is same everywhere) while weight changes from place to place.
- Mass is measured using a beam balance while weight is measured using a spring
balance
- Mass is a scalar quantity while weight is a vector quantity.

− 0−500 −500
B2 (a) = = = = − / .
0.2 0.2

Therefore, deceleration = 2500m/s2


50
(b) F = ma = 1000 × −2500 = -125N
B3 (a) Velocity ratio (V.R) = 2
. .
(b) = × 100 → 75 = × 100 → 100 . = 75 × 2
. 2
100 . 75×2 150
= → . =  M.A = 1.5
100 100 100
120
(c) . = → = = = 80N
. 1.5

(d) First calculate the distance moved by effort

. = → 2=
0.5

Distance moved by effort = 0.5 x 2 = 1m


Now calculate work done by effort
Work done by effort = force x distance = 80N x 1m = 80J
B4 (a) (i) Frequency = 10Hz (because frequency is number of waves per second).
(ii) V = fƛ  30 = 10 x ʎ  30 = 10ʎ  ʎ = 3m
(b) Loudness depends on amplitude of the sound wave.
B5 (a) ∠r = 90 – 65 = 25o
sin 1 sin 50 0.7660
(b) = sin = = = 1.81
sin 25 0.4226

(c) Medium B is denser than medium A


B6 (a)
(b) The image is real.
The image is inverted.
The image is the same size as the object.
ℎ ℎ 1
(c) = = =1
ℎ ℎ 1

× 220 ×400
B7 (a) = = = 800 turns
110

1100
(b) (i) = = = 10A
110

× 110 ×10
(ii) = = = 5A
220

1 × 2 6 ×6 36
B8 (a) = = = = 3.0 Ω
1+ 2 6+6 12

6
(b) = = = 2A
3

(c) Power = Voltage x Current = 6 x 2 = 12W


B9 (a) Nucleon number = 131
24
(b) = (1) = = → =
0 2 8

1 1 3
=( ) −→ 1
= (2) −→ =
0 2 4 ×108 4 ×108 8
4 ×108
× 8 = 1 × (4 × 108) −→ 8 = 4 × 108 → =
8
7
N = 5 x 10 per second.

SECTION C [20 MARKS]


C1 (a) (i) Absolute zero temperature is the temperature at a substance has
lowest kinetic energy which is equal to -273K and at this temperature
molecular motion ceases.
(ii) Charles’ Law states that the volume of an ideal gas at constant pressure
is directly proportional to the absolute temperature.
(b) (i)
Temp (oC) 17 28 39 60 80 100
Volume (cm3) 7.5 7.9 8.1 8.7 9.1 9.6
Temp (K) 290 301 312 333 353 373
(ii)

(iii) Volume of a gas when Temperature = 77oC (350K) = 9.1cm3


C2 (a) To absorb radiate heat and radiates it back to the water pipe.
(b) To prevent heat loss through conduction and radiation.
(c) Through conduction and convection

1000 1000 ×2
(d) = ( )= × (2 2) = =2000W
1 1

2000
Power = 1000 = 2Kw

60
(e) = 100 × 2 = 1.2

3
= × (2.5 2) = 5cm2
1.2

C3 (a) Aluminium
(b)
Date Mass of original radioactive substance left (kg)
1 July 2008 8
1 July 2012 4
1 July 2020 1

(c) (i)
Time in years 0 1 2 3 4 5 6
Recorded activities in counts/min 124 80 52 34 23 16 12
Activity due to sample alone 120 76 48 30 19 12 8
Table C3.2
(ii) Background radiation is a radiation from the surroundings.
TITLE: Activity due to sample alone against time

1.2+1.8 3
(iv) Half – life = = = 1.5 years
2 2
ANSWERS FOR SCIENCE PAPER 1 – 2020 INTERNAL
SECTION A [20 Marks]
1. D 5. B 9. B 13. A 17. D
2. D 6. D 10. D 14. A 18. A
3. A 7. C 11. C 15. D 19. A
4. C 8. A 12. C 16. A 20. B

SECTION B [45 Marks]

1 (a) Mass of a pendulum bob


16.1 + 15.9 + 16.0 + 16.2 + 15.8 80
(b) = = = 16
5 5
16
= = = 0.8 seconds
20
1 1
(c) = = = 1.25Hz
0.8

30
2 (i) V = u + at  30 = 0 + 1.2 x t  = 1.2 = 25 seconds

(ii) Force = mass x acceleration = 900 x 1.2 = 1080N

3 (a) (i) Weight = mass x gravity = 80 x 10 = 800N


(ii) F1 x d1 = F2 x d2  800 x 0.5 = F2 x 2
800 × 0.5 400
F2 = = = 200N
2 2

(b) Force, F, would reduce.

4 (a) Potential energy = 72 joules


72
(b) . = = 36J
2
. 72 72
(c) ℎ ℎ = = = = 3.6m
2 × 10 20

(d) = √2 ℎ = √2 × 10 × 3.6 = √72 = 8.485m/s


5 (a) Because it has compression region and rarefaction region
Because its vibration particle is parallel to direction of the wave.
(b) (i) Region U : Compression
(ii) Region W: Rarefaction
(c) Because in liquid particles are closely packed than in gas.
Because liquid is denser than gas.

6 (i) angle of incident (i) = 40o and angle of refraction (r) = 29o
sin sin 40 0.643
= = = = 1.3259 = 1.33
sin sin 29 0.485
1 1
(ii) sin = → = = 0.751879699
1.33

= −1(0.751879699) = 48.7534. Therefore, Critical angle = 48.75o

7 (a) (i) Total power = 120 W + 40 W + 40 W = 200W


200
(ii) Energy supplied = 1000 × 3 = 0.2 x 3 = 0.60kWh

(iii) Potential difference = 240V because voltage in parallel is equally distributed.

8 (a)

(b)
(c)

Deflection of
electron beam

9 (a) A beta particle is a radiation whose nature is like that of an electron with 0 mass and
negative (-1) charge.
32 0
(b) (i) 15 → −1 + Therefore,
Mass number Proton number
32 = 0 + A 15 = -1 + Z  Z = 15 + 1
A = 32 Z = 16

0
(ii) 32
15 → 2 −1 +  → − +

(c) Point radiation sources (phosphorous – 32) away from people.


Wear lead lined clothing (lead absorbs more radiation than most other materials).
Wear gloves and use tongs when handling substances that emit radiation.
SECTION C [20 Marks]
C1 (a) (i) Between O and P – The motor cycle started from rest and accelerated
uniformly by increased its velocity to 40m/s in 3 seconds.

(ii) Between P and Q – The motor cycle moved with a constant velocity of 40 m/s
for 4 seconds.

(iii) Between Q and R - the motor cycle retardate to rest uniformly from 40m/s to 0
m/s in 5seconds.

(b) Maximum speed = 40m/s

(c) It took 5 seconds

1 1
(d) Distance = ( + )ℎ = (4 + 12) 40 = 16 × 20 = 320m
2 2

320
(e) = = = 26.67m/s
12

C2 (a)
Mass / kg 0 0.02 0.04 0.06 0.08 0.10
Length of loaded spring / cm 11 12.1 13.2 14.3 15.4 16.5
Applied force / N 0 0.2 0.4 0.6 0.8 1.0
Extension /mm 0 11 22 33 44 55
TITLE: Force (N) against Extension (mm)

2− 1 0.6 − 0.2 0.4


(c) The gradient = Spring constant = = = = 0.01818 = 0.02N/mm
2− 1 33 − 11 22

(d) (i) NO, It did not


(ii) Because extension against force gives a straight-line graph passing through the
origin.
C3 (a) (i)
Gamma X – rays Ultraviolet Blue Green Infra – red Microwave Radio
rays waves

(ii) 1. They are all transverse waves.


2. They transfer energy from one place to another.
3. They all show wave properties such as reflection and refraction.
4. They can all travel through a vacuum.
5. They all travel at 3 x 108ms-1 in a vacuum.
6. The equation v = fλ applies to all of them.

(b) (i) Radio wave


(ii) Infra – red radiation

3 ×108
(c) (i) ℎ= = = 0.03m
1010

(ii) Microwave is used in microwave oven


Microwave is used in radar
Microwave is used in telecommunication
ANSWERS FOR SCIENCE PAPER 1 – 2021 G.C.E

SECTION A [20 MARKS]

1. X 6. B 11. C 16. A
2. X 7. A 12. C 17. D
3. A 8. D 13. C 18. D
4. B 9. D 14. C 19. C
5. C 10. B 15. D 20. C

SECTION B [45 MARKS]


B1 (a) Main reading: 5.30 cm
Vernier reading: 3 x 0.01 = 0.03 cm
Final reading: 5.30cm + 0.03 cm = 5.33cm

5.33
(b) (i) 5.33cm into metres = = 0.0533m
100

(ii) 5.33 x 10-2m


(i) Three significant figures

B2 (a) Mass = 75kg


(b) Weight = mg = 75kg x 1.6N/kg = 120N
(c) Force = mass x acceleration = 2000kg x 1.5m/s2 = 3000N

B3 (a) For a body to be in equilibrium (balanced), the sum of


clockwise moments about any point is equal to the sum of
anticlockwise moments about the same point.
(b) F1 x d1 = F2 x d2
2× 2 4.0 ×40
1= =
20
= 8N
1

(c) Newton metre reading will decrease because the acting of force two (F2) is
closer to the pivot.
B4 (a) Work = mass x gravity x height
Work = 0.14 x 10 x 12 = 16.8J

(b) V = √2 ℎ = √2 × 10 × 12 = √240 = 15.49m/s


(c) Because wind acts as air resistance and created friction force which reduced the
resultant force on the stone.

B5 (a) Number of waves (n) = 5 waves


(b) Amplitude = 5m
6
(c) Speed = = = 3m/s
2

B6 (a) Refractive index is the ratio of the sine of the angle of


incidence to the sine of the angle of refraction.
60
(b) (i) = −→ 1.6 =
60
1.6 × = 60 −→ =
1.6
0.86603
= = 0.54126875
1.6

Angle r = sine-1 (0.54126875) = 32.77o

(ii) Critical angle = −1 (1) = −1 (


1
) = 38.68o
1.6

B7 (a) (i) Magnetic field is the region around a magnet where


magnetic effect can be detected.
(ii) Electric field is the region of space where an electric charge experiences
a force due to other charges is.
(b)
(c)

B8 (a)

16
(b) = = = 2A
8

(c) Potential difference = Current x Resistance = 2 x 6 = 12V

B9 (a) Nucleon number is the sum total number of protons and


neutrons in an atom.

238 → 234 ℎ + 4
(b) 92 90 2

234 ℎ → 0
(c) 90 ℎ + −1

A = 234 - 0 = 234

Z = 90 - (-1) = 90 + 1 = 91

234 ℎ → 234 ℎ + 0
90 91 −1
SECTION C [20 MARKS]
C1 (a) Mass its S.I unit is kilogram (kg) while weight its S.I unit is
Newton (N).
Mass is the quantity of matter in a substance while weight is the pull of gravity
on an object.
(b) (i) Weight = mg = 20 x 10 = 200N
(ii) P.E = mgh = 20 x 10 x 6 = 1200J
20 ×10×9
(c) = = = 150W
12

(d) E= Pt = VIt = 230V x 1.5A x 0.003.33 = 1.15Wh

60
C2 (a) = −→ 1.50 =
60
1.50 × = 60 −→ =
1.50
0.86603
= = 0.577350269
1.50

Angle r = sine-1 (0.577350269) = 35.26o


(b) Critical angle = −1 (1) = −1 (
1
) = 41.81o
1.50

(c) Because the ray of light will pass from less dense medium to high denser
medium and the speed of light ray reduces hence the it bend.

(d) (i) ℎ =
300 000 /
= = 123966.9421km/s
2.42

(ii) Critical angle = −1 (1) = −1 (


1
) = 24.41o
2.42

C3 (a) (i) Because they have alpha radiation which cannot


penetrate the human body.
(ii) 241 →
4 + +
95 2

Value of X = 241 – 4 = 237


Value of Y = 95 – 2 = 93
241 →
4 + 237 +
95 2 93
( )( )

1+ 2+ 3 450+475+375
(ii) Half –life = = = 433.33years
3 3

(c) Because it has small half – life. It can be finished decaying within short period of
time of which large quantity is needed or replacement time to time.
THE END
ANSWERS FOR SCIENCE PAPER 1 – 2021 INTERNAL
SECTION A [20 MARKS]
1. D 6. X, (18Nm) 11.B 16.D
2. D 7. B 12.C 17.B
3. A 8. C 13.B 18.D
4. B 9. D 14.D 19.D
5. C 10.D 15.A 20.C

SECTION B [45 MARKS]


B1 (a) Total distance = 60 km
(b) Time = 3h – 2.2h = 0.8 hours
Distance 30
(c) Speed = = = 13.64km/h
Time 2.2
Total distance 58
(d) Average speed = = 5
= 11.6km/h
Total time
(e) Causes accidents

B2 (a) Newton’s second law of motion states that an unbalanced force


acting on a body produces an acceleration in the direction of the
force which is directly proportional to the force and inversely
proportional to the mass of the body.
Force (16+12)−10
(b) Acceleration = = = 4.5m/s2
Mass 4

B3 (a) Useful energy = 300J


(b) Wasted energy = 1000 – (300 + 70 + 30) = 600J
Energy output 300
(c) Efficiency = × 100% = × 100% = 30%
Energy input 1000

B4 (a) Convection is the process by which heat is transmitted from one


place to another by the movement of heated particles of a liquid or a
gas.

(b) (i) Land breeze


(ii)

B5 (a) Radio waves, Microwaves, Infra-red radiation (IR), Visible light, Ultra-
violet,radiation (UV), X-rays, Gamma (ϒ) rays.

(b) 1. They are all transverse waves.


2. They transfer energy from one place to another.
3. They all show wave properties such as reflection and refraction.
4. They can all travel through a vacuum.
5. They all travel at 3 x 108 ms-1 in a vacuum.
6. The equation v = fλ applies to all of them.
(c) 1. They all have different wavelength.
2. They all have different frequency.
(d) 1. Gamma causes cancer
2 Ultra-violet causes sun – tan and skin cancer.

B6 (a) Ultrasound mean sounds with frequencies greater than 20 000 Hz.
(b) Thickness of Z = Speed x time = 4100 x (9.0 x 10 – 6) = 0.0369m = 3.69 x 10 – 2m
(c) Ultrasound is used for detecting flaws in metal joints.
Ultrasound is used for cleaning jewellery
Ultrasound is used for sonar instruments.
B7 (a) Reflection of light is the bouncing back of light ray as it strike an object.
(b)

(c) 1. Same size as object (hi = ho)


2. Laterally inverted
3. Erect
4. Virtual
5. As far behind the mirror as the object is in front (v = u)

B8 (a) Current = 3.0A


voltage 1.0V
(b) Resistance = = = 1.0 Ω
current 1.0A
(c) Power = voltage x current = 6 x 3 = 18.0 W

B9 (a) Nuclear fission is the splitting of a nucleus of an atom due to neutron


bombardment to release a large amount of energy.
(b) (i) Fission fragments
235 1 144 90
(ii) 92U + 0n → 56Ba + 36 Kr + 210n
(c) Fission reaction produces huge amount of energy (heat energy)
which can destroy (damage) buildings and animal life.
It produces some deadly products like plutonium – 239 which can kill
anything breathed in dust.

(d) It produces heat energy at the stead rate.


SECTION C [20 MARKS]

C1 (a) To determine the lower fixed point of a thermometer,


place the thermometer bulb in melting ice. The mercury
will contract hence the mercury thread will be lowered.
Then measure the length of the mercury thread and mark
its lower fixed point.

(b) 1 Mercury has a high boiling point (357 oC) while alcohol It
has a low boiling point (78 oC).
2 Mercury is silvery coloured and doesn’t allow light
through (It is easy to see) while alcohol is colourless
but is made visible by adding coloring.
3 mercury does not wet glass. (It doesn’t stick to glass)
while alcohol stick to the wall of the capillary tube
when the thread is falling.

(c) (i) Sensitivity of a thermoment – make the bulb large and


narrower the capillary tube.
(ii) Accuracy of the thermomenter – Shorten the range of it.
(d) (i) Thermocouple (Resistance thermometer)
(ii) 1 Gas turbine exhaust
2 Diesel engines
3 Temperature sensor
4 Flame sensor

C2 (a)
U (cm) 30.0 35.0 40.0 45.0 50.0
V (cm) 44.0 37.0 32.0 31.0 28.0
V
M( ) 1.47 1.06 0.80 0.69 0.56
U

(c) Co-ordinates are (31, 0.69) and (36.80, 1.04)


y2 − y1 1.04 −0.69 0.35
Slope (G)=
x2 − x1
= = = 0.06
36.80 − 31 5.8
28 −
(d) = − 1 → 0.06 = 28 − 1 → 0.06 = → 28 − =
0.06
28
+ 0.06 = 28 → 1.06 = 28 → = = 17.5cm
1.06
(e) It is used in eye glasses, microscope, camera and projectors.
C3 (a) Hooke’s Law states that the extension of a loaded spring is directly
proportional to the load (force) applied.
(b) (i)
Mass (kg) 0 0.2 0.4 0.6 0.8
Pointer reading (mm) 120 126 132 138 144
Force (N) 0 2 4 6 8
(c) (ii)
FORCE (N) 0 2 4 6 8
EXTENSION (mm) 0 6 12 18 24

(b) (iii) Extension = 15mm


(c) Elasticity is the ability of a material to resume its normal shape after
being stretched or compressed while plasticity is the quality of being
easily shaped or molded.
Elasticity deformation is reversible while plasticity deformation is
irreversible.

You might also like